HS326
Which of the following statements is not correct regarding stock bonus plans? A) Stock bonus plans allow for the current deductibility of non-cash contributions. B) The required repurchase option for a stock bonus plan can create potential cash flow issues in the future. C) Stock bonus plans are generally as cost efficient to operate as profit-sharing plans or money purchase pension plans. D) The eligibility for a stock bonus plan could be age 20 and six months of service.
Stock bonus plans are generally as cost efficient to operate as profit-sharing plans or money purchase pension plans. Options A, B and D are true. Option C is incorrect as the cost to value stock for a private stock bonus plan increases the administrative costs in comparison to profit-sharing or money purchase plans.
Jay died in a freak accident. Fortunately for his girlfriend Toni there was a whole life insurance policy on Jay's life as part of his qualified plan at work. At the time of Jay's death: Policy face value was $25,000 with a cash value of $11,000. Jay had paid income taxes on $1,000 of premiums over the years. Taxable portion of death benefit =
Taxable portion of death benefit: $11,000 - $1,000 = $10,000.
Marleen is a 52-year-old participant in the XYZ cash-balance plan. She has been a participant in the plan for the last 20 years. XYZ, which has over 10,000 employees, is having financial difficulty and Marleen is concerned about the security of her pension. Which of the following is correct? Question 5 options: A) The cash-balance plan assets may include up to 25 percent of XYZ stock. B) The cash-balance plan formula cannot be lowered in the future for current participants, but it could be changed for new participants. C) The cash-balance plan and its benefits are fully protected by the Pension Benefit Guaranty Corporation (PBGC). D) Termination of the plan may affect vesting for some employees, but Marleen's vesting will not be affected.
Termination of the plan may affect vesting for some employees, but Marleen's vesting will not be affected. Option (A) is incorrect because up to 10 percent of plan assets can be invested in the plan sponsor's stock. Option (B) is incorrect as the plan sponsor cannot modify future benefits. Option (C) is incorrect because the PBGC does not fully protect all benefits; it only protects benefits up to an annual limit. Option (D) is correct; termination accelerates vesting, but based on Marleen's tenure, she is fully vested already.
Mindy, who is 44 years old, has five IRAs. On January 12th, she converts $40,000 in IRA 2, which is a traditional IRA, into a Roth IRA. On March 25th, she takes a distribution of $20,000 from IRA 1. On May 20th, she rolls over the $20,000 into IRA 4. Which of the following statements is correct? A) Mindy owes tax and penalty on $40,000. B) Mindy owes tax and penalty on the $20,000 because it represents her second rollover during the year. C) The $20,000 rollover qualifies for the 60-day rollover exception. D) Mindy owes ordinary income tax on $60,000, but no penalty.
The $20,000 rollover qualifies for the 60-day rollover exception. The first distribution is a Roth conversion and is subject to income tax, but no penalty. The second distribution qualified for the 60-day rollover exception, which means it is not subject to income tax or penalty.
Ruth was laid off by her employer ZZZ inc. at age 56. While struggling to find work she made a $15,000 withdrawal from her account in the 401(k) plan sponsored by ZZZ. What taxes will Ruth owe on this withdrawal?
The 401(k) withdrawal will be subject to tax at ordinary income rates. But because she separated from service after age 55, there will be no 10% penalty.
Considering the use of an in-plan Roth rollover to convert assets in a qualified plan to a Roth subaccount, which of the following statements is true? The conversation may result in a reduction in income tax in future years. The conversion will eliminate the need for minimum distributions during the participant's lifetime. Only participants whose adjusted gross income (AGI) is below a maximum threshold are permitted to do an in-plan Roth rollover. An in-plan Roth rollover is also referred to as a "back-door Roth."
The conversation may result in a reduction in income tax in future years. Funds within a qualified plan's Roth subaccount may be distributed tax-free, subject to certain limitations. The second statement is incorrect because funds within a qualified plan are still subject to required minimum distributions even if they are in a Roth subaccount. The third statements is incorrect because the AGI maximum applies only to contributions to a Roth IRA, not a Roth subaccount within a qualified plan. The fourth statement is incorrect because the back-door Roth concerns non-deductible contributions to a Traditional IRA which is then converted into a Roth IRA.
From the perspective of an employer, which of the following is a disadvantage of sponsoring a qualified retirement savings plan? Employer contributions provide an income tax deduction. Sponsoring such a plan helps attract and retain employees. The legal requirements are often complex. Employer contributions are not subject to payroll tax.
The legal requirements are often complex. From an employer's perspective, the legal complexities involved in sponsoring a qualified plan are often significant and constitute a disadvantage. However, this disadvantage is generally outweighed by the advantages offered to the employer by sponsoring such a plan.
All of the following are prohibited transactions, EXCEPT: The plan invests in a new business venture of the plan administrator. The plan lends money to a sponsoring company's 100 percent owner The plan invests funds in 30-year U.S. treasury bonds. The union for covered employees receives financial support from the plan.
The plan invests funds in 30-year U.S. treasury bonds The plan is free to invest assets in publicly traded securities if the plan investment advisor determines that the securities are appropriate given the plan's Investment Policy Statement. The first option is prohibited because the plan administrator is a disqualified party. The second option is prohibited because no lending is allowed to a party in interest. The fourth option is prohibited because an employee organization which covers any plan participants is a disqualified person.
All of the following are true with regards to the use of life insurance in qualified plans EXCEPT: Question 12 options: A) Whole Life insurance premiums cannot exceed 50% of the sponsor's aggregate contribution to a participant's defined contribution account. B) Universal Life insurance premiums cannot exceed 25% of the sponsor's aggregate contribution to a participant's defined contribution account. C) Life insurance can only provide an incidental death benefit in a qualified plan, with the exception of a 412(e)(3) plan. D) There is no basis accumulation for any Table 2001 amounts imputed to a plan participant for life insurance held by a qualified plan for their benefit.
There is no basis accumulation for any Table 2001 amounts imputed to a plan participant for life insurance held by a qualified plan for their benefit. Statements (A), (B), and (C) are correct. There is basis accumulation for any Table 2001 amounts paid for the cost of pure term insurance by a plan participant.
ISOs are a common benefit for executives. Which of the following statements is not correct regarding ISOs? A) ISOs can only be granted to an employee of the corporation issuing the ISOs. B) The exercise of the ISO is limited to a 10-year period. C) To the extent that the aggregate fair market value of stock with respect to which ISOs are exercisable for the 1st time by any individual during any calendar year exceeds $100,000, such options shall be treated as NQSOs. D) To qualify as an ISO, the executive must hold the stock for either two years from the grant of the ISO or one year from the date of exercise of the ISO.
To qualify as an ISO, the executive must hold the stock for either two years from the grant of the ISO or one year from the date of exercise of the ISO. All statements are correct except D. The statement in answer D was changed from "the greater of" to "either."
Which of the following is the most common defined-benefit plan funding formula for large companies? Question 3 options: A) Flat amount formula B) Flat percentage formula C) Unit credit formula D) Excludable amount formula
Unit credit formula The most common funding formula for a defined benefit plan is the unit credit formula, which provides higher benefits for longer term employees. The formula in option D does not exist.
Heidi has received a qualified stock bonus from her employer, Trinitron. She wants to know what rights this provides her. Which of the following is a feature Trinitron must provide Heidi as a feature in the stock bonus plan? The right to attend Board of Director's meetings Voting rights as a result of owning stock through the plan A monthly valuation of the stock by 30 days after the end of the month The right to purchase additional stock on the same terms (up to a maximum)
Voting rights as a result of owning stock through the plan One of the features of a qualified stock bonus plan is that the employee receiving the stock must have voting rights. None of the other features are part of a stock bonus plan
Erin received 1,000 nonqualified stock options (NQSOs) with an exercise price of $25 per share when the stock was $25 on the market. Two years from the date of grant, Erin exercises the stock when the stock price is $102. At exercise, Erin has W-2 income of $25,000. an alternative minimum tax (AMT) adjustment of $77,000. W-2 income of $77,000. an AMT adjustment of $25,000.
W-2 income of $77,000. Erin will have W-2 income of the difference between the market price and the exercise price. ($102 − $25) × 1,000 = $77,000 She will not have an AMT adjustment for the exercise of an NQSO.
All of the following are examples of investments that may be held in an IRA, EXCEPT: Tesla stock SPDR gold shares ETF IBM corporate bond Whole life policy
Whole life policy
Declan, aged 46, is a self-employed financial planner and has Schedule C income of $56,000 from self-employment. He has not saved for retirement until now. Therefore, he would like to make the maximum contribution to his profit-sharing plan. How much can he contribute to his profit-sharing plan account this year? $9,464 $10,409 $11,200 $14,000
$10,409 Self-employment tax = ($56,000 x 92.35%) x 15.3% = $7,913 ½ Self-employment tax = $3,956 Earned Income = $56,000 - $3,956 = $52,044 Maximum "employer" contribution = 20% x $52,044 = $10,409
Nicki is a highly skilled sales person at Byberry, which is a 30-year-old company that has grown significantly in terms of revenue and product offerings. It sponsors a pension plan that provides a benefit of 2% times the years of service times the average of the final three years of salary. Nicki has worked for Byberry for the last 30 years and earned $200,000 two years ago, $150,000 last year, and $250,000 this year. If she is retiring this year, how much should she expect to receive as a pension benefit? Question 7 options: A) $120,000 B) $160,000 C) $185,000 D) $200,000
$120,000 The average salary over the last three years is $200,000. The benefit equals 60 percent of $200,000 or $120,000.
Corey has a vested account balance in his employer-sponsored qualified money purchase pension plan of $60,000. He has two years of service with his employer and the plan follows the least generous graduated vesting schedule permitted under PPA 2006. If Corey has an outstanding loan balance within the prior 12 months of $15,000, what is the maximum loan Corey could take from this qualified plan, assuming the plan permitted loans? A) $15,000 B) $30,000 C) $35,000 D) $50,000
$15,000 The maximum loan an individual can take is the lesser of $50,000 or 50% of their vested account balance. In this case, Corey has a vested account balance of $60,000, so the maximum loan would be 50% of $60,000, or $30,000. However, since Corey had an outstanding loan balance of $15,000 within 12 months, the maximum loan available must be reduced by $15,000.
Jim's employer offers a defined-benefit pension whose payout is a function of the 3 highest years of earnings. Jim currently has a 3-year average earnings of $70,000 and his accrued monthly pension is $1,500. If Jim's employer wishes to add an incidental whole life insurance policy to the pension plan, what is the maximum death benefit that can be offered to Jim? $150,000 $350,000 $700,000 $1,500,000
$150,000 A qualified pension plan is limited in the amount of life insurance it is able to purchase with plan assets. Either the 25%/50% test or the 100-to-1 ratio test must be passed for the insurance to be considered incidental. Since we have no information about aggregate employer contributions, it isn't possible to apply the 25% / 50% test. Using the 100-to-1 ratio test the maximum death benefit is 100 x $1,500 = $150,000.
Kay turned 73 on March 17th of Year 2 (which was after the year 2023 and before the year 2033). Her profit-sharing account balance was $500,000 at the end of Year 1 and $550,000 at the end of Year 2. Her beneficiary is her favorite granddaughter, Jordan, who turned 12 years old on July 23rd of Year 2. Assume that the joint life expectancy factor for a 73-year-old and a 12-year-old is 73 and the joint life expectancy for a 74-year-old and a 13-year-old is 72. Also, assume that the life expectancy factor based on the uniform lifetime table for someone who is 72, 73 and 74, is 27.4, 26.5, and 25.5, respectively. Kay takes a distribution of $10,000 in November of Year 1 and in Year 2. What is the Kay's minimum distribution for Year 2? A) $6,849. B) $18,868. C) $20,073. D) $20,755.
$18,868. The Year 2 required minimum distribution for Kay is $18,867.92 ($500,000 divided by 26.5) because she is 73 years old as of December 31 of Year 2. The amount of the distribution is not relevant, nor is her beneficiary's age, unless it would have been a spouse more than ten years younger. She has no distribution required for Year 1 because the first distribution must be made by April 1st of the year following the attainment of age 73.
Nancy turned age 72 on November 2nd, 2022 and had the following account balances in a qualified retirement plan: 12/31/2021 $519,000 12/31/2022 $600,000 Assuming that Nancy is retired and has never taken a distribution from her plan, what is the minimum distribution required by April 1st, 2023? Life expectancy factors according to the uniform life table are 27.4 for a 72 year-old and 26.5 for a 73 year-old. $18,038 $18,941 $20,273 $23,438
$18,941 Because Nancy turned 70 after July 1st, 2019, she is subject to the SECURE ACT RMD rules. Due to changes made by the SECURE Act, if your 70th birthday is July 1, 2019, or later, you do not have to take withdrawals until you reach age 72. This is the same for qualified plans and IRAs. Roth IRAs do not require withdrawals until after the death of the owner. The calculation is as follows: For 2022, look back to 2021: $519,000/27.4 = $18,941 If she waits till April 1st, 2023, her Required Beginning Date (RBD), to take her 2022 distribution, Nancy will have to take another distribution by December 31st, 2023 for her 2023 RMD.
Halley has worked for GT for the last 20 years and been a participant in its defined benefit plan. In the last ten years, her salary has increased significantly. Over the last ten years, her compensation was $290,000, $100,000, $120,000, $100,000, $240,000, $200,000, $160,000, $180,000, $150,000, and $210,000. In 2023, what is the most that she could receive as a pension payment? Question 9 options: A) $200,000 B) $265,000 C) $246,667 D) $285,000
$200,000 The correct answer is (A).The maximum distribution from a defined benefit plan in 2023 is the lesser of $265,000 or the average of the three highest consecutive years of compensation. The average of the three highest consecutive years of compensation equals $200,000.
Bailey owns and operates Bailey's Red Truck Shop (BRT), which is a sole proprietorship. She has selfemployment income of $150,000. How much self-employment tax does she owe for 2023? A) $21,194.33. B) $21,527.10 C) $22,057.20. D) $22,950.00.
$21,194.33. 150,000 x 0.9235 = (A)138,525 Max Social Sec Wage Base = (B) 160,200 Lesser of A & B = 1138,525 OASDI (amount above x 12.4% (0.124) = 17,177.10 A x 2.9% (0.029) = 4017.23 17,177.10 + 4017,23 = 21,194.33
Tobias, aged 42, has compensation of $72,000. The normal retirement age for his 457(b) plan is age 62. Tobias has unused deferrals totaling $22,500 as of the beginning of the year. How much can Tobias defer into his 457(b) public plan for 2023?
$22,500 Tobias is not within 3 years of the plan's normal retirement age and therefore can only defer the normal $22,500 (2023). The $7,500 catch up (2023) for those participants who are 50 years old or older is not available because he is only 42 years old.
Catherine, aged 42, earns $300,000 annually as an employee for DUN, Inc. Her employer sponsors a SIMPLE retirement plan and matches all employee contributions made to the plan dollar-for-dollar up to 3 percent of covered compensation. What is the total contribution (employer and employee) that can be made to Catherine's SIMPLE account in 2023? $22,000 $24,500 $25,000 $26,000
$24,500 The maximum employee contribution for 2023 is $15,500. The employer has chosen to make matching contributions of up to 3 percent of compensation (the SIMPLE maximum). Therefore the employer can make a contribution of up to $9,000, or $300,000 compensation × 3%. Added together we have $24,500 that can be contributed to her account
Last year, Mark elected to defer $25,000 from his wages under his company's deferred compensation plan. Which of the following statements is correct as to the treatment of the $25,000? $25,000 will be included in Mark's gross income this year. $25,000 will be subject to payroll taxes last year. $25,000 will be included in Mark's gross income when it is paid out to him in the future. Mark will be responsible for payroll taxes on the $25,000 when it is paid out to him in the future.
$25,000 will be included in Mark's gross income when it is paid out to him in the future. The $25,000 will be subject to payroll taxes this year and will be included in Mark's gross income when it is paid out to him in the future, not in the year of the deferral.
Drake has worked for GT for the last 20 years and been a participant in its defined benefit plan. In the last ten years, his salary has increased significantly. Over the last ten years, his compensation was $300,000, $145,000, $200,000, $400,000, $225,000, $240,000, $233,000, $210,000, $150,000, and $290,000, respectively. In 2023, what is the most that he could receive as a pension payment? Question 8 options: A) $265,000 B) $285,000 C) $288,333 D) $330,000
$265,000 The maximum distribution from a defined benefit plan in 2023 is $265,000.
Jeri is a 56-year-old human resources specialist for Preston Foundries. She wants to maximize her contributions to her company's retirement plans. Based on her $80,000 salary, she was allocated $45,000 to her profit-sharing plan for the year. Preston Foundries does not match employee deferrals to their 401(k) plan. What is the maximum amount Jeri can defer into her 401(k) plan in 2023? $22,500 $25,500 $28,500 $66,000
$28,500 In 2023, her maximum deferral to the 401(k) is $22,500 plus the age 50 or older catch-up deferral of $7,500, for a total of $30,000. However, the deferral is also included in the maximum defined-contribution limit of $66,000 (2023; not including the catch-up provision). Since Jeri has received an allocation from the profit-sharing plan of $45,000, she is able to defer $21, 000 ($66,000 − $45,000) plus the $7,500 catch-up deferral for participants who are 50 years old and older for a total of $28,500.
Brady, age 51, earns $333,333 annually from Infinity, which has 20 employees and sponsors a SIMPLE IRA. Infinity matches all employee deferrals 100% up to a 3% contribution. What is the maximum total contribution to Brady's account in 2023 and 2024 (assume the indexed numbers for 2023 remain the same in 2024 and that Brady's salary also remains the same), including both employee and employer contributions? A) $15,500 in 2023 and $17,050 in 2024. B) $19,000 in 2023 and $20,900 in 2024. C) $25,500 in 2023 and 25,500 in 2024. D) $29,000 in 2023 and $30,900 in 2024.
$29,000 in 2023 and $30,900 in 2024. Brady can defer up to $19,000 ($15,500 + $3,500) for 2023 because he is age 50 or older. The match for Brady is 3% of his compensation, or $10,000 (3% x $333,333). The maximum contribution to Brady's SIMPLE is $29,000 ($15,500 + $3,500 + $10,000) in 2023. Under the SECURE 2.0 Act, beginning in 2024, the regular contribution and catch-up limits are increased by ten percent for employers with 25 or fewer employees. Since the question states that the regular contribution and catch-up limits in 2024 are assumed to be the same as in 2023 ($15,500 and $3,500 respectively), the contribution limits for Brady will be $15,500 x 1.10 = $17,050 and $3,500 x 1.10 = $3,850. The maximum contribution to Brady's SIMPLE in 2024 is $30,900 ($17,050 + $3,850 + $10,000).
On January 5, Sheila, aged 39, withdrew $42,000 from her qualified plan. Sheila had an account balance of $180,000 and an adjusted basis in the account of $30,000. What is her early withdrawal penalty? $0 $1,200 $3,500 $4,200
$3,500 $30,000 ÷ $180,000 = 0.1667 exclusion $42,000 × 0.1667 = $7,000 $42,000 − $7,000 = $35,000 $35,000 × 0.10 = $3,500 early withdrawal penalty
EFG Corp provides employees with discounts on the flat-panel televisions they manufacture. The discounts were established using a length-of-service and employee-status methodology. Length of Service - Employee Discount - Officer Discount 1 - 5 years - 10% - 20% 6-10 years - 15% - 30% 11+ years - 20% - 40% The gross profit percentage for EFG is 40 percent. Andy is an officer employee who has been with EFG Corp for 13 years. For Hanukkah this year, Andy bought a 56-inch flat-panel television that retails for $8,800, and he received a discount appropriate to the schedule of discounts listed above. For this year, how much must Andy include in gross income as a result of this transaction? $0 $1,760 $2,640 $3,520
$3,520 The plan is clearly discriminatory; therefore, Andy, who is an officer, must include the entire discount of $3,520 ($8,800 x 0.40).
Whitney is covered by a $90,000 group term life insurance policy, and her daughter is the sole beneficiary. Whitney's employer pays the entire premium for the policy; the uniform premium is $0.66 per $1,000 per month of coverage. How much income would be included in Whitney's W-2 for the year? $0 $26.40 $316.80 $712.80
$316.80 $50,000 of group term life insurance is nontaxable. $90,000 − $50,000 = $40,000 ($40,000 ÷ $1,000) × $0.66 × 12 = $316.80 included in W-2 income
Evelyn, age 51, is employed by Carlisle Company as a marketing consultant. She earns $55,000 per year and always maximizes her contributions to the company 401(k) plan. Rather than matching employee 401(k) deferrals, Carlisle Company profit-sharing plan contributes at the end of the year. What is the maximum profit-sharing contribution that can be made for Evelyn at the end of 2023? $28,000 $31,000 $32,500 $35,500
$32,500 he maximum combined value of employee and employer contributions is the lower of $66,000 or 100 percent of compensation. For Evelyn, this maximum is thus $55,000 (100% of her compensation). Therefore, the maximum profit-sharing contribution is $32,500, or $55,000 − $22,500. The catch-up of $7,500 would not have mattered in this calculation as it is not considered for 415(c) calculations.
Mark owns MN, Ltd., which is a professional firm with five employees and that sponsors both a defined-benefit plan and a profit-sharing plan with a cash or deferred arrangement (CODA). The covered compensation for MN, Ltd., is $600,000. Salary deferrals total $30,000. If the required funding for the defined-benefit plan is $120,000, then how much can be contributed to the profit-sharing plan? $0 $30,000 $36,000 $150,000
$36,000. Salary deferrals are not taken into consideration for the multiple-plan limits. Since the plan is not subject to the Pension Benefit Guaranty Corporation (PBGC) because MN, Ltd., is a professional firm with less than 25 employees, the combined limits will apply unless the defined contribution does not exceed 6 percent. The funding for the defined-contribution plan is the greater of the remaining 25 percent limit after taking into consideration the defined-benefit funding or 6 percent. Therefore, $36,000, or 6% of $600,000 can be contributed to the defined-contribution plan since it is greater than $30,000, ($600,000 x 25%) $120,000.
Kendra has an account balance in her employer's money purchase pension plan of $100,000. The plan has a 2-to-6-year graded vesting policy. She has been a participant for three and a half years and has worked for the company for five years. Assuming the plan permits loans, what is the maximum loan that Kendra could take from the plan? A) $20,000 B) $30,000 C) $40,000 D) $50,000
$40,000 The maximum loan an individual can take is the lesser of $50,000 or 50% of their vested account balance. In this case, Kendra is 80% vested because she has worked for five years, which equates to 80% on the 2-to-6-year graded schedule. Half of 80% of $100,000 equals $40,000.
Amber, who is 49 years old, received a distribution from her Roth account of her employer's 401(k) plan in the amount of $100,000 on August 11th. She has been a participant in the plan for ten years. Her adjusted basis in the plan was $600,000 and the fair market value of the account as of August 11 was $1 million. The distribution was for the purpose of buying a Porsche for herself for her birthday. What is the taxable amount of the distribution and any applicable penalty? A) $0 taxable, $0 penalty because it is a qualified distribution. B) $40,000 taxable, $4,000 tax penalty. C) $40,000 taxable, $0 tax penalty. D) $100,000 taxable, $10,000 tax penalty.
$40,000 taxable, $4,000 tax penalty. The distribution is not a qualified distribution because she is not age 59½, disabled or dead. Thus, the pro rata portion of the distribution that relates to the earnings is taxable - 40% in this case. The penalty is 10% of $40,000 or $4,000.
Ethan, who is 45 years old, is having a few leadership issues and must resign from his current position as CEO. He worked for A-Send, which sponsors a standard 401(k) plan. The plan uses the longest permitted vesting schedule and is not top heavy. He has deferred $40,000 into the 401(k) plan and has employer- matching contributions of $20,000. If he has been employed for 2 1/2 years, but has only participated in the plan for the last 2 years, how much does he keep if he resigns and terminates his employment today? $44,000 $60,000 $124,000 $140,000
$44,000. Vesting is based on years of service with the employer. Ethan keeps his entire deferral of $40,000. The matching contributions will vest over a 2- to-6-year graded schedule, which means that he keeps 20 percent of the $20,000 for a total of $44,000.
Donna turned 74 on January 7th of 2025. Her profit-sharing account balance was $100,000 at the end of 2024 and $150,000 at the end of 2025. Her beneficiary is her older sister, Robin, who turned 82 years old on July 2nd of 2025. Assume that the life expectancy factor based on the uniform lifetime table for someone who is 73, 74, and 75 is 26.5, 25.5 and 24.6, respectively. If Donna only takes a distribution of $2,000 for 2025, then how much is her minimum distribution penalty (assume that the failure is not corrected during the correction window)? A) $480.39. B) $960.78. C) $980.39. D) $1,921.57.
$480.39. Balance (end of 2024) $100,000 Factor 25.5 Minimum Distribution $3,921.57 Distribution $2,000 Difference $1,921.57 25% Penalty $480.39
Bright Sun is a 30-year-old company that has grown significantly in terms of revenue and product offerings. It sponsors a pension plan that provides a benefit which can be calculated as (2 percent × years of service × the average of the final 3 years of salary) − a Social Security integration offset. The offset equals 1 percent times years of service times income below the covered compensation limit of $40,000 (assumed). Lorenzo has worked with Bright Sun for the last 30 years and earned $90,000 2 years ago, $110,000 last year, and $130,000 this year. If he is retiring this year, how much should he expect to receive as a pension benefit? $45,000 $54,000 $66,000 $78,000
$54,000 Average Salary$110,000 Benefit ($110,000 × 2% × 30 years)$66,000 Less Offset ($40,000 × 1% × 30 years)- $12,000 Actual Benefit $54,000
On March 1st, Ian became employed by a large corporation that has 400 employees. The corporation provides its employees with a no-cost gym membership to a local YMCA. The cost of the membership is $60 per month, which is completely paid for by Ian's employer. How much must Ian include in this year's gross income related to this no-cost fringe benefit? $0 $60 $600 $720
$600 In order for the value of the athletic facilities to be excluded from an employee's gross income, the athletic facilities must be located on the employer's business premises and must be for the exclusive use of the employees and their dependents. Ian's membership is at a public YMCA, so it is taxable. Because he started with for the corporation in March, he received 10 months of the benefit. 10 × $60 = $600
Larissa, who is age 45, has just resigned from her current job. She worked for Ace, which sponsors a cash balance plan and a standard 401(k) plan. Each of the plans uses the longest permitted vesting schedule and both plans are top heavy. Larissa has a balance of $40,000 in the cash balance plan, has deferred $20,000 into the 401(k) plan and has employer matching contributions of $10,000. If she has been employed for three years, but only participating in the plans for the last two years, how much does she keep if she leaves today? A) $20,000. B) $30,000. C) $60,000. D) $64,000.
$64,000. Vesting is based on years of service with the employer. Larissa keeps the entire deferral of $20k. The cash balance plan uses a 3-year cliff, which means that she keeps all of the $40k. The matching contributions will vest over a 2-to-6-year graded schedule, which means that she keeps 40% of the $10k, for a total of $64k.
Gareth sells stock 6 months after he received it as a distribution from a qualified stock bonus plan. When the stock was distributed, he had a net unrealized appreciation (NUA) of $7,500. He also had ordinary income from the distribution of $29,000. The fair market value (FMV) of the stock at the time of sale was $81,000. How much of the sale price will be subject to long-term capital-gain treatment? $7,500 $44,500 $52,000 $73,500
$7,500 Appreciation on the stock after the date of distribution is taxed as long-term capital gain or short-term capital gain, depending on the holding period beginning at the date of distribution. In this case, only the net unrealized appreciation of $7,500 is treated as long-term capital gain because the holding period for the sale was only 6 months. The remaining $44,500 of gain is taxed as short-term capital gain. $81,000 FMV − $29,000 ordinary income = $52,000 total capital gain $52,000 − $7,500 NUA long-term capital gain = $44,500 short-term capital gain
Sierra, aged 62, single, and retired receives a defined-benefit pension annuity of $1,200 per month from Mancini Corporation. She is currently working part-time for Imani's Interior Design and will be paid $18,000 this year (2023). Imani's Interior Design has a 401(k) plan, but neither Sierra nor Imani's Interior Design has made a contribution to the plan this year. What is the maximum contribution that Sierra can contribute to a traditional IRA or a Roth IRA in 2023? $6,500 to a traditional IRA or $6,500 to a Roth IRA $0 to a traditional IRA or $6,500 to a Roth IRA $7,500 to a traditional IRA or $0 to a Roth IRA $7,500 to a traditional IRA or $7,500 to a Roth IRA
$7,500 to a traditional IRA or $7,500 to a Roth IRA Sierra has earned income and is over 50 years old. She is not an active participant in a retirement plan, and even if she were, her adjusted gross income (AGI) is below the income limits. The maximum contribution limits for IRAs and Roth IRAs are $6,500 if a person is younger than 50 and $7,500 if a person is 50 years old or older.
Urban LLC sponsors a profit-sharing plan that requires employees to complete one year of service and be 21 years old before entering the plan. The plan also excludes all commissioned sales people and all other allowable exclusions allowed under the IRC. Which of the following employees could be excluded? Jax, age 20, who works in administration and has been with the company for 32 months. Gemma, a commissioned sales person working in the Houston office. She is 37 years old and has been with the company for 3 years. Clay works as the lead foreman in the company factory. He is 39 and has been with the company for 12 years and is covered under a collective bargaining agreement. A) 1 only. B) 3 only. C) 1 and 3. D) 1, 2, and 3.
1, 2, and 3. Each of these employees can be excluded from the plan. Jax does not meet the age requirement. Gemma can be excluded because she is a commissioned salesperson. Clay is excluded because he is covered under a collective bargaining agreement.
Sheehan works for Andy Company and is a superior sales guy. His total compensation this year is $400,000. Andy sponsors an integrated profit-sharing plan with a base percentage of 5% and a maximum excess percentage. It uses the current wage base as the integration level. What is the excess percentage that will be applied to Sheehan's compensation above the integration level? A) 5.0%. B) 5.7%. C) 10.0%. D) 10.7%.
10.0%. The excess percentage is 10% - it is limited to twice the base percentage.
Fred, aged 45, has just now worked long enough to become eligible to participate in his employer's stock bonus plan. Fred learns that each year his employer will contribute 15 percent of his compensation into his plan account in the form of company stock. Fred will also need to work for his employer for 6 years to be fully vested in these contributions. Because Fred understands the value of portfolio diversification, he would prefer that his account assets were invested in a mixture of bond and stock funds. How many more years will Fred need to work for his employer before he will have the right to diversify his account holdings? 1 year 2 years 3 years 5 years
2 years Stock bonus plans are required to allow participants to diversify employer contributions after 3 years or more of service, but in order to become a plan participant, Fred first needed to satisfy the age-21-and-1-year-of-service eligibility requirement that is common to all qualified plans employing 2-to-6 year graduated vesting. Since he is 45 years old and has just become eligible to participate, this means that he has currently completed just 1 year of service. After 2 more years of service, he will have satisfied the 3 year requirement and will be able to diversify his account holdings out of 100 percent company stock.
What is the first year in which a single taxpayer who is 54 years old in 2020 could receive a qualified distribution from a Roth IRA if he opened a Roth IRA for the first time on April 1, 2021 for the tax year 2020? 2023 2024 2025 2026
2025 A qualified distribution can only occur after a 5-year period has occurred and if it is made on or after the date on which the owner attains age 59½, is made to a beneficiary or the estate of the owner on or after the date of the owner's death, is attributable to the owner being disabled, or it is for a first-time home purchase. The 5-year period begins at the beginning of the taxable year of the initial contribution to a Roth IRA. The 5-year period ends on the last day of the individual's fifth consecutive taxable year from the taxable year described in the preceding sentence. Therefore the first year in which a qualified distribution could occur is 2025.
A top-heavy profit-sharing plan may use any of the following vesting schedules EXCEPT: 1-to 4 year graduated. 35 percent after 1 year, 70 percent after 2 years, and 100 percent after 3 years. 2-to-6 year graduated. 4-year cliff.
4-year cliff. The only choice that is not possible is a 4-year cliff. A 3-year cliff is the standard for a defined contribution plan. Top-heavy status is irrelevant with defined-contribution plans after the Pension Protection Act of 2006 (PPA 2006).
Seth, who is about 55 years old, runs a local Po'Boy shop in New Orleans. He has several high school kids who work for him part-time. Seth's mom, Robin, helps make the best meatballs in the universe and works there full-time. Which of the following plans makes the most sense for Seth if he earns about $120,000 and does not want to spend too much on a retirement plan? A) Money purchase plan. B) Tandem plan. C) Cash balance plan. D) 401(k) / profit-sharing plan.
401(k) / profit-sharing plan The first three options require annual funding, which is generally not the best choice for a small business. Option d permits Seth to save over $50,000 per year or almost half of his income, between the salary deferral, catch-up and profit-sharing contribution. In addition, he has no requirement to fund it all, as the plan is discretionary.
Generally, which of the following plans are considered contributory plans? 401(k) and money purchase pension plans 401(k) and thrift plans Thrift plans and employee stock ownership plans (ESOPs) Money purchase pension plans and profit-sharing plans
401(k) and thrift plans A contributory plan is a plan that requires the plan participant to contribute to the plan. While employers generally contribute to money purchase pension plans, ESOPs, and profit-sharing plans, employees contribute to 401(k) and thrift plans
Jacob is a participant in JJ's defined benefit plan. Jacob is 37 years old and earns $160,000. He has 4 years of service for purposes of the plan and has worked at the firm for 5 years. The plan provides a benefit of 1.5% for each year of participation. The plan has the least generous vesting schedule possible. Almost 70 percent of the accrued benefits are attributable to the fifteen equal owners, who have all been working at the company for decades. If Jacob were to leave today, what percent of his salary (as defined by the plan) could he expect to receive at normal retirement age? Question 6 options: A) 3.6% B) 4.8% C) 6.0% D) 6.4%
6.4% This plan is top heavy based on the 70% of benefits for the equal owners - they each own more than 5% and are key employees. Therefore, vesting will shift and there is a minimum benefit that must be provided to non-key employees. His benefit will be 2% times YOS (8%) up to 20%. However, the benefit must be reduced based on his vesting. Five years of service represents 80% vesting. The benefit equals (8% x 80%) = 6.4%
Nautilus, Inc., recently adopted a profit-sharing plan. Nautilus has two offices, the North Nautilus office and the South Nautilus office. There are ten employees in the North Nautilus office, five of whom are eligible for the plan. There are 15 employees in the South Nautilus office, all of whom are eligible for the plan. Which of the following statements is true with regard to the profit-sharing plan? If Nautilus decides to notify the employees about the plan via mail, the letters must be mailed at least 30 days before the determination letter is mailed to the IRS. A Summary Plan Description must be furnished to each participant within 120 days of plan establishment. If Nautilus adopted a prototype plan, then Nautilus will use a single trust or custodial account that has been adopted by all employers using that prototype plan. Nautilus only needs to notify the employees eligible for the plan that the company ha
A Summary Plan Description must be furnished to each participant within 120 days of plan establishment. A Summary Plan Description must be furnished to each participant within 120 days of plan establishment. The first option is incorrect because the employees must be notified by mail between 10 and 24 days before mailing the determination letter. The third option describes a master plan. The fourth option is incorrect because all employees must be notified if they work in an office where an eligible employee exists.
Which of the following pension plans has mandatory Pension Benefit Guaranty Corporation (PBGC) coverage? 403(b) plan sponsored by a public school system A defined benefit plan sponsored by a five-person attorney group A money purchase plan sponsored by a retail chain A cash balance plan sponsored by a large chemical company
A cash balance plan sponsored by a large chemical company Only qualified plans are eligible for PBGC coverage, so the 403(b) plan is ineligible. Although defined-benefits plans are generally covered, those sponsored by professional service employers with 25 or fewer employees are excluded. Defined-contribution plans such as a money purchase plan are also excluded.
Which of the following statements regarding defined-benefit plans is true? A defined-benefit plan can allocate forfeitures to other plan participants. A defined-benefit plan can use forfeitures to reduce future plan costs. A defined-benefit plan cannot give credit for prior service. Each participant of a defined-benefit plan has an individual account.
A defined-benefit plan must use forfeitures to reduce future plan costs. Statement (A) is incorrect because a defined benefit plan cannot allocate forfeitures to other plan participants. Statement (C) is incorrect because a defined benefit plan can give credit for prior service. Statement (D) is incorrect because participants in a defined benefit plan do not have individual accounts.
Which of the following distributions from a profit-sharing plan would not be subject to the 10 percent early withdrawal penalty, assuming the participant has not attained age 59½? A) A distribution made from an IRA that was established for a 50-year old spouse under a qualified domestic relations order (QDRO) pursuant to a divorce from the participant in the profit-sharing plan. B) A distribution from the plan to a participant who lost his right leg above the knee in a car accident. C) A distribution to pay for costs of higher education for a participant's 18-year-old daughter. D) A distribution made to the participant after she separated from service at age 57.
A distribution made to the participant after she separated from service at age 57. Option a is subject to the 10% penalty because the distribution is from the IRA and not from the profitsharing plan. Option b is subject to the 10% penalty because loss of one limb is not likely to be considered a disability in terms of the early withdrawal penalty. Option c is subject to the 10% penalty because education is not an exception for distributions from a qualified plan.
Dave, age 40, made a $10,000 withdrawal from his employer's profit sharing plan. His account contained $15,000 of after-tax contributions and $45,000 of employer contributions and earnings. What taxes will Dave owe on this withdrawal?
A pro rata portion is tax-free: $10,000 x ($15,000 / $60,000) = $2,500. The remaining $7,500 will be subject to income tax at ordinary rates, as well as a $750 penalty tax on the early withdrawal.
Which of the following statements about employer choices among profit-sharing plans is correct? Only publicly traded companies can employ age-based profit-sharing plans. A safe harbor 401(k) plan simplifies plan administration. Every employee must receive the same dollar amount of profit-sharing contribution. New comparability plans are inexpensive to create and simple to administer.
A safe harbor 401(k) plan simplifies plan administration. One of the main advantages of a safe harbor 401(k) plan is that it can automatically satisfy ADP, ACP, and top-heavy testing requirements, thus simplifying plan administration. Answer (A) is incorrect because there is no restriction that would allow only publicly traded companies to use age-based plans. Answer (C) is incorrect because the standard allocation formula for a profit-sharing plan assigns equal percentages of compensation, not equal dollar amounts, to each participant. Answer (D) is incorrect because new comparability plans are actually complicated and expensive to establish and administer.
Charles, a single 29-year-old, deferred 2% of his salary, or $2,500, into a 401(k) plan sponsored by his employer during 2023. What is the maximum deductible IRA contribution Charles can make during 2023? Question 61 options: A) $0 B) $1,000 C) $4,000 D) $6,000
A) $0 Charles cannot make a deductible IRA contribution for the year because he is an active participant in a qualified plan with an AGI of at least $125,000 ($2,500 / 2%), which exceeds the phase-out limits for 2023.
James is the property manager of a large apartment complex where he lives at the request of his employer. While he lives there for free, similar units rent for $2,000 per month on annual leases and $1,500 per month on two-year leases. How much must he include in his gross income for living on the premises of his employer? Question 46 options: A) $0 B) $1,000 per month C) $1,500 per month D) $2,000 per month
A) $0 James is required by the employer to live on the premises and therefore will not include the value of the lodging in his gross income.
Robin works for a small publishing company. The company built a 2,500-square-foot gym on its premises. The cost of the facility was $80,000. The cost of a comparable membership to a local gym is $60/month. Robin, her husband, and her two children all use the gym regularly. How much, if anything, must Robin include in her yearly gross income related to this fringe benefit? A) $0 B) $720 C) $2,160 D) $2,880
A) $0 Robin does not have to include any income for the use of the athletic facilities; the exclusion for payment of health club facility dues is only provided when the facilities are on the employer's business premises and are solely for the use of the employees and their family members. Thus, this use qualifies. TCJA 2017 did not change this provision.
Natalie is a secretary at JKL Law Firm. JKL provides her with free sodas at her discretion. Natalie estimates that she drinks $20 worth of sodas per month. How much must Natalie include in her annual gross income related to the sodas? Question 88 options: A) $0 B) $20 C) $200 D) $240
A) $0 The value of employer-provided de minimis fringe benefits may be excluded from an employee's gross income. After 2017, however, the employer's deduction for the sodas is limited to 50 percent due to TCJA 2017.
Crystal, a waitress at Hot Dog Heaven, works from 7 a.m. to 4 p.m. five days a week. Each workday she is furnished, without charge, two meals, valued at $10 each. The manager of Hot Dog Heaven encourages her to eat breakfast in the employee breakroom each day before 7 a.m. but does not expressly require her to do so. The manager does, however, require her to eat her lunch in the employee breakroom. How much of the value of her meals does she have to include in her gross income per day? A) $0 B) $10 C) $15 D) $20
A) $0 These meals are provided for the convenience of the employer. The changes under TCJA 2017 should not impact this scenario as this likely falls into the category of de minimis.
Maria, age 40, is a member of Marpro, LLC. Marpro sponsors a profit-sharing plan. Maria's portion of the net income was $60,000, and one-half of her self-employment taxes were $4,239 for 2023. If Marpro makes a contribution of 25 percent of salary on behalf of all of its employees to the profit-sharing plan, how much is the contribution to the profit-sharing plan on behalf of Marpro for Maria? Question 55 options: A) $11,152 B) $13,940 C) $22,500 D) $30,000
A) $11,152 $60,000 (net income) − $4,239 (1/2 of self-employment tax) = $55,761 net self-employment income. $55,761 × (0.25/1.25) = $55,761 × 0.20 = 11,152
Reggie has attained 2 years of service with his employer, Ink, Inc., (INK). INK sponsors a profit-sharing plan that provides for loans, and Reggie's account balance within the plan is $150,000. He has not contributed any money to the profit-sharing plan. If the plan follows the least generous graded vesting schedule permitted under IRS guidelines, what is the maximum loan Reggie can take considering that he has not previously taken any plan loan? Question 23 options: A) $15,000 B) $30,000 C) $75,000 D) $150,000
A) $15,000 The maximum loan permissible is the lesser of 50% of the participant's vested account balance or $50,000, reduced by the highest outstanding loan balance within the 12 months prior to taking the new plan loan. In this case, Thomas is 20% vested in his profit-sharing plan account balance because he has only attained 2 years of service with the organization. (The least generous graded vesting schedule permitted would be a 2-to-6 year graded vesting schedule.) Since he is only vested in 20% of the account balance ($30,000), 50% of that vested account balance would be $15,000. Since Thomas has not taken any previous loans, his maximum loan would not be adjusted any further.
Julius owns and operates Julius's Exotic Journeys (JEJ), a sole proprietorship. JEJ sponsors a profitsharing plan. Julius had net income of $150,000 and paid self-employment taxes of $20,000 (assumed) during the year. Julius has decided to make a 15% contribution for his employees for the year. AssumingJulius is over the age of 50, what amount will he contribute for himself to the plan for 2023? A) $18,261. B) $21,000. C) $28,000. D) $25,761.
A) $18,261. $150,000 Net Income ($10,000) Minus ½ SE Tax $140,000 Net SE Income × 0.130435 (0.15 / 1.15) $18,261 Note: $21,000 represents 15% of the net SE income. This is not correct. $28,000 represents 20% of the net SE income and would be the maximum contribution assuming everyone received a 25% contribution. $25,761 is the profit sharing contribution of $18,261 plus $7,500 catch-up; however, the catch-up contribution is only available for employee contributions to a 401(k) plan; therefore, it is not available toJulius.
Shake Shack's defined-benefit pension plan uses a funding formula that considers years of service multiplied by 2.5% and then by highest average compensation for 3 of the last 5 years of service to determine the pension benefit payable to the plan participants. Emily is a participant in Shake Shack's defined-benefit pension plan and has 30 years of service with the company. Her salary has been $200,000, $200,000, $250,000, $250,000, $250,000 a year for the last 5 years, respectively with Shake Shack. What pension benefit could Emily expect to receive per year from her defined benefit plan at her retirement? Question 13 options: A) $187,500 B) $200,000 C) $230,000 D) $250,000
A) $187,500 Emily has 30 years of qualifying service which would be multiplied by 2.5% to arrive at a multiple of 75% of her final highest average salary for 3 of the last 5 years of her employment. In that time frame, Emily has 3 years of salary averaged at $250,000 which would be her highest 3-year average. Emily therefore would be offered 75% of $250,000, which is $187,500 per year.
Eric, aged 45, is self-employed and has net income of $105,000; one-half of his self-employment taxes were $7,419 for 2023. If he sponsors a profit-sharing plan and makes a contribution of 25% of salary to the profit-sharing plan on behalf of all of its employees, how much is the contribution to the profit-sharing plan on behalf of Eric? Question 30 options: A) $19,516 B) $22,500 C) $24,395 D) $26,250
A) $19,516 $105,000 (net income) − $7,419 (1/2 of self-employment tax) = $97,581 net self-employment income. $97,581 × (0.25/1.25) = $97,581 × 0.20 = 19,516
Spencer owns and operates Spencer's Sales Solutions (3S), a sole proprietorship. 3S sponsors a profitsharing plan. Spencer had net income of $250,000 and paid self-employment taxes of $22,000 (assumed) during the year. Assuming Spencer is over the age of 50, what is the maximum amount that he can contribute to the profit-sharing plan on his behalf for 2023? A) $47,800 B) $50,000 C) $55,300 D) $59,750
A) $47,800 $250,000 Net Income ($11,000) Minus ½ SE Tax $239,000 Net SE Income × 0.20 = 0.25/1.25 $47,800 Option B is 20% of the net income of $250,000. Option C includes the $7,500 catch-up contribution. The fact that Spencer is over the age of 50 is not relevant. Catch-up contributions are not made by employers, only by employees to CODAs. Option D is 25% of the net self-employment income of $239,000.
Leteisha's Law Practice (LLP), has decided to install both a Defined Benefit (DB) and a Profit-Sharing Plan/401(k) Plan to max out tax advantaged retirement savings in the last few years of Leteisha's working career. Leteisha, age 60, is the only General Partner in the practice and does not have to include her other limited partner, Jay, who is a 20% owner. Her annual compensation for Plan purposes is $500,000. In advising Leteisha and in coordination with the DB Plan actuary you are working with, you tell her that she needs to make a $250,000 contribution for the DB Plan this year. For the PS/401K Plan, the covered compensation for all the participants in the plan is $1,000,000. In the DC Plan, there is no match for employee contributions but there has been a 6% of compensation non-elective profit share every year and Leteisha tells you that she can fund that again this year. Given the fact pattern above, what is th
A) $48,300 With regards to Leteisha as an employee of her own business, since she is over 50 she will be able to add $22,500 plus a $7,500 catch-up for a total of $30,000 of her own money. With regards to employer contributions, since the Plan does not exceed a 6% of compensation contribution, LLP can exceed multiple plan limitations which would have restricted contributions to the greater of the funding amount required for our defined benefit plan or 25% of covered compensation of those participating in the plan $250,000 in our example). 6% of the Leteisha's compensation, reduced from $500,000 to $330,000 is $18,300. If we add $18,300 + $30,000 we arrive at $48,300.
Vance has a vested account balance in his qualified profit-sharing plan of $40,000. He has two years of qualifying service with his employer. The plan follows the least generous graduated vesting schedule permitted for a profit-sharing plan. If Vance has an outstanding loan balance of $15,000 within the prior 12 months, what is the maximum loan amount Vance could take from his qualified plan? Question 44 options: A) $5,000 B) $20,000 C) $40,000 D) $50,000
A) $5,000 The maximum loan an individual can take is the lesser of $50,000 or 50% of their vested account balance. In this case, Vance has a vested account balance of 20% (2 years of service with a two-to-six year vesting schedule) of $200,000, or $40,000; so the maximum loan would be 50% of $40,000, or $20,000. Since Vance had an outstanding loan balance of $15,000 within 12 months, the maximum loan available must be reduced by $15,000. In this case, the maximum loan Vance could take from the qualified plan is $5,000 ($20,000 − $15,000).
Marleen, who is 37 years old, is an employee of Zcrypt, Inc. (Zcrypt). Zcrypt sponsors a SEP IRA and would like to contribute the maximum amount to Marleen's account for the plan year. If Marleen earns $32,000 per year, what is the maximum contribution Zcrypt can make on her behalf to the SEP IRA? A) $8,000 B) $19,000 C) $32,000 D) $57,000
A) $8,000 Contributions to a SEP IRA are limited to the lesser of 25 percent of the employee's compensation or $66,000 (2023). In this case, Marleen's compensation is $32,000, so the contribution on her behalf would be limited to 25 percent of $32,000, or $8,000.
Acme Inc. has 200 total employees, 150 of which are nonexcludable employees. Ten employees are highly compensated. Seven of the 10 highly compensated and 100 of the 140 nonhighly compensated employees are covered under Acme's qualified plan. The average accrued benefits for the highly compensated is 3% and the average accrued benefit for the nonhighly compensated is 1.5%. Which of the following statements is true regarding coverage? The plan passes the ratio percentage test. The plan passes the average benefits test. A) 1 only B) 2 only C) Both 1 and 2. D) Neither 1 nor 2
A) 1 only The ratio percentage test compares the % of nonhighly compensated to the % of highly compensated covered. The ratio must be greater than or equal to 70% for the plan to pass the ratio percentage test. The calculation for Acme's qualified plan is as follows: NHC = 100/140 = 71.43% (pass the General Safe Harbor Test which is not asked) HC = 7/10 = 70% 71.43%/70% = 102% (pass) Acme's plan passes the ratio percentage test requirement of 70%. The average benefits test requires the average benefit of the nonhighly compensated employees to be at least 70% of the average benefit of the highly compensated. Acme's plan does not satisfy the average benefits test because the average benefit of the nonhighly compensated compared to the average benefit of the highly compensated is less than 70% (1.5%/3% = 50%) (fail).
Which of the following are correct regarding 403(b) plans? Employee elective deferrals may be contributed to a Roth account within a 403(b) plan. 403(b) plans must comply with the ADP and ACP tests. An employee of the New Orleans Museum of Art (a not-for-profit organization) could defer morethan $30,000 into the Museum's 403(b) plan. A) 1 only. B) 1 and 2. C) 1 and 3. D) 2 and 3.
A) 1 only. Choice 2 is incorrect as 403(b) plans do not have to comply with the ADP test. Choice 3 is not correct as the only way to exceed $30,000 (2023) would be to make use of the additional catch-up contribution, which is not available to an employee of a museum - only health, education or religion.
SEPs and SIMPLEs are tax advantaged retirement plans that are less complex than qualified plans and easier to implement. Which of the following statements is (are) true regarding SEPs and SIMPLEs? For an employee with a salary of $30,000, more money can be contributed (from both employee and employer) to a SEP than a SIMPLE. The vesting and distribution rules for both plans are almost identical. Both plans have the same coverage and participation rules. A) 2 only. B) 1 and 2. C) 2 and 3. D) 1, 2, and 3.
A) 2 only. Statements 1 is not correct as the deferral into a SIMPLE is greater than 25% of the $30,000 salary, without regard to a catch-up and/or a match by the employer. Statement 2 is correct. SIMPLEs do have the extra penalty for two years, but other than that they are the same. Statement 3 is not correct. SIMPLEs have a $5,000 earnings eligibility threshold, SEPs have a $750 (in 2023) eligibility threshold.
Cheque Company has 100 eligible employees and sponsors a defined-benefit pension plan. The company is unsure if it is meeting all of the testing requirements. How many employees (the minimum) must be covered by Cheque Company's defined-benefit pension plan for the plan to conform with ERISA? Question 16 options: A) 40 B) 50 C) 70 D) 100
A) 40 The 50/40 rule requires that defined-benefit plans cover the lesser of 50 employees or 40% of all eligible employees. In this example, 40% of 100, or 40 employees, would be the lesser of these two amounts.
All of the following statements about SEP and SIMPLE IRAs are true EXCEPT: Question 70 options: A) A SEP requires the plan sponsor to provide at least a 100% match up to 3% of all employee deferrals. B) A SEP plan can be established by employers who employ more than 100 employees who earn $5,000 or more during the preceding calendar year. C) SIMPLEs can be either contributory or noncontributory plans, whereas SEP plans are always noncontributory. D) An employer who wants to share the responsibility of retirement plan funding should establish a SIMPLE rather than a SEP.
A) A SEP requires the plan sponsor to provide at least a 100% match up to 3% of all employee deferrals. Statement (A) is incorrect because a SEP is a noncontributory plan that does not receive employee deferral contributions. Statement (B) is correct as the 100 employee limit applies to SIMPLEs. Statement (C) is correct as SIMPLEs can be either contributory or noncontributory and SEPs are noncontributory. Statement (D) is correct because a SIMPLE can be a contributory plan, thus receiving allocations from employee deferrals, and sharing the burden of plan funding with the employee.
Of the following statements regarding target-benefit pension plans, which is true? Question 21 options: A) A target-benefit pension plan is a money-purchase pension plan with a funding formula that actuarially considers age and salary. B) The plan sponsor of a target-benefit pension plan guarantees that the participant will receive an amount, expected to be the "target benefit" amount, at his retirement. C) Plan participants of a target-benefit pension plan can choose to invest up 20% of their account assets into employer stock. D) The plan sponsor guarantees an earnings rate of at least 6 percent for the contributions made to target-benefit pension plans.
A) A target-benefit pension plan is a money-purchase pension plan with a funding formula that actuarially considers age and salary. Statement (A) is true. Statements (B), (C), and (D) are false. The plan sponsor is required to fund the participant of a target-benefit pension plan's separate account each year with the actuarially equivalent present value of the "target." The "target" is the expected benefit that the plan sponsor forecasts (at inception) that the participant will receive at retirement, but the sponsor does not make any guarantees on the benefit payable from the plan. Statement (C) is incorrect because no qualified plan may invest more than 10% of its assets into employer securities. Statement (D) is incorrect because the plan sponsor does not guarantee a minimum investment return on participant account assets.
IRC §132(e) defines a fringe benefit as any property or service provided to an employee by an employer that is so small in value that it makes accounting for it unreasonable or administratively impracticable when taking into account the frequency with which similar fringe benefits are provided by the employer to employees. What type of fringe benefit is this? A) De minimis B) Working-condition fringe C) Employee benefit D) No-additional-cost fringe
A) De minimis B) No. It does not exceed the $400 limit.
Steve is self-employed as a marketing consultant. He works primarily with start-up internet companies helping to develop corporate brand programs. Several years ago, he established a 401(k) profit-sharing plan and has accumulated $385,000 in the plan. Which of the following forms should he file to meet his compliance requirements? Question 54 options: A) Form 5500 EZ B) Form 5500 SF C) Form 5500 D) He does not need to file a Form 5500 of any type.
A) Form 5500 EZ He must file Form 5500 EZ since it is a one-participant plan and total assets exceed $250,000. If assets were below this threshold, he would not have to file the form.
Sebastian operates a small business with his wife Ariel. They have no other employees. They have $340,000 of assets in the plan. Which of the following forms must they file? A) Form 5500-EZ. B) Form 5500-SF. C) Form 5500. D) They do not have to file form 5500.
A) Form 5500-EZ. Because the plan only covers the owner and spouse, they can file Form 5500-EZ.
Which of the following statements concerning HSAs is correct? Question 100 options: A) HSA distributions that are used to pay for non-medical expenses are subject to a 20% penalty. B) Any HSA distributions not used for medical expenses are subject to long-term capital gains tax rates. C) If HSA contributions are less than the prescribed limits, the account earnings are included in the account holder's gross income for federal income tax purposes. D) A married couple may contribute to up to one medical FSA and up to one HSA in any given year.
A) HSA distributions that are used to pay for non-medical expenses are subject to a 20% penalty. HSA distributions used to pay for non-medical expenses are subject to a 20% penalty as well as federal income tax (not, as statement B states, long-term capital gains taxes). Statement (C) is incorrect because, under the circumstances described, the account earnings would not be included in the account holder's gross income for federal income tax purposes. Statement (D) is incorrect because if one spouse is covered by an FSA, this is treated as both spouses are being covered. If eligible, a married couple must select between an FSA and an HSA. An exception is if the FSA is "limited purpose" (covers dental and vision only).
Ron was awarded 5,000 shares of restricted stock at a time when the stock price was $10. Assume Ron properly makes an 83(b) election at the date of the award. The stock vests 5 years later at a price of $20, and he sells it then. What are the tax consequences in the year he makes the 83(b) election? Question 40 options: A) He has W-2 income of $50,000. B) He has a long-term capital gain of $50,000. C) He has W-2 income of $10,000. D) He has a long-term capital gain of $10,000.
A) He has W-2 income of $50,000. At the time Ron makes the 83(b) election, the value of the stock at that date will be included in his taxable income. Thus, Ron will have W-2 income of $50,000 (that is, $10 × 5,000).
Generally, when must employees include the value of a fringe benefit in their gross income? I. When they were given the choice to decline the fringe benefitII. When the benefit is provided for the convenience of the employer Question 90 options: A) I only B) II only C) Both I and II D) Neither I nor II
A) I only Generally, employees must include the value of a fringe benefit in their gross income when they were given the choice to decline the benefit but took it anyway. For example, an employee who was given the option (but not the obligation) to live in employer-provided lodging would likely be required to include the value of that lodging in his or her gross income. Statement II is incorrect because benefits provided for the convenience of the employer are generally not included in the employee's gross income.
Which of the following is true regarding negative elections? I. A negative election is a provision whereby the employee is deemed to have elected a specific deferral unless the employee specifically elects out of such election in writing.II. Negative elections are no longer approved by the IRS.III. When an employer includes a negative election in its qualified plan, the employer must also provide 100% immediate vesting. Question 30 options: A) I only B) I and III C) II and III D) I, II, and III
A) I only Negative elections are approved by the IRS, and they are available for both current and new employees. Negative elections do not require 100% immediate vesting.
The beneficiary(ies) of a key-person life insurance policy generally is (are) I. the employer II. the key person's spouse or dependents A) I only B) II only C) Both I and II D) Neither I nor II
A) I only The beneficiary is usually the company because the company has suffered the loss of the key person. The key person's family would need to purchase a separate life insurance policy.
Wanka Factory has 100 non-excludable employees, 10 of whom are highly compensated. Eight of the 10 highly compensated and 63 of the 90 non-highly compensated employees are covered under Wanka's qualified plan. The average accrued benefits for the highly compensated is 4% and the average accrued benefit for the non-highly compensated is 1.5%. Which of the following statements is true regarding coverage? I. The plan passes the ratio percentage test.II. The plan passes the average benefits percentage test. Question 11 options: A) I only B) II only C) Both I and II D) Neither I nor II
A) I only The ratio percentage test compares the % of covered non-highly compensated to the % of covered highly compensated employees. The ratio must be greater than or equal to 70% for the plan to pass the ratio percentage test. The calculation for Wanka's qualified plan is as follows: NHC = 63/90 = 70% HC = 8/10 = 80% 70%/80% = 87.5% Wanka's plan passes the ratio-percentage test requirement of 70%. The average benefits percentage test requires the average benefit of the non-highly compensated employees to be at least 70% of the average benefit of the highly compensated. Wanka's plan does not satisfy the average benefits test because the average benefit of the non-highly compensated compared to the average benefit of the highly compensated is less than 70% (1.5%/4% = 37.5% = fail).
The taxonomy of plan selection includes a number of steps. The initial step is to determine which type of plan to select. If the employer wants to consider a qualified plan, which of the following requirements must be met: I. Coverage rules II. Vesting rules III. Testing rules IV. Eligibility rules Question 28 options: A) I, II, III, IV B) I, II, III C) II, III, IV D) I, II, IV
A) I, II, III, IV All four listed rules are requirements that must be met in order to install a qualified retirement plan in a business or institution.
Which of the following is (are) elements of an effective waiver for a qualified preretirement survivor annuity? I. The waiver must be signed within six months of death. II. The waiver must be signed by a plan participant. III. The waiver must be notarized or signed by a plan official. A) III only B) I and II C) II and III D) I, II, and III
A) III only Statement III is correct. Statement II is incorrect, as only the nonparticipant spouse must sign the waiver. Statement I is unfounded.
All of the following statements concerning IRA planning considerations are correct EXCEPT: Question 68 options: A) If a participant's marginal tax bracket is higher at the time of the contribution than at the time of the distribution, making a Roth IRA contribution is preferred to making a deductible IRA contribution. B) A Roth IRA typically provides better tax benefits than a nondeductible traditional IRA. C) Contributing to a 401(k) plan instead of contributing to a Roth IRA may result in a greater accumulation of wealth if the employer matches the 401(k) salary deferral. D) A Roth IRA conversion can be a good estate-planning tool for those who will be subject to estate or inheritance taxes.
A) If a participant's marginal tax bracket is higher at the time of the contribution than at the time of the distribution, making a Roth IRA contribution is preferred to making a deductible IRA contribution. When marginal tax rates are higher in the year of contribution than they will be in the year(s) of distribution, making a deductible IRA contribution can provide additional tax savings and allow for a greater accumulation of wealth as compared to making a Roth IRA contribution.
All of the following statements concerning a qualified plan's summary plan description (SPD) are correct EXCEPT: Question 57 options: A) It typically is used to persuade employees to join the plan. B) It is an easy-to-read booklet that explains the plan to the participants. C) It may be prepared by the financial services professional, the insurer, or the employer. D) It must be issued to participants within 120 days after the plan is adopted by the board of directors.
A) It typically is used to persuade employees to join the plan. The SPD cannot be used as a sales piece. The other statements are correct.
All the following types of income are considered earned income for the purposes of making a traditional IRA contribution EXCEPT A) K-1 income from an S corporation. B) W-2 income. C) self-employment income. D) alimony resulting from a divorce agreement signed June 1, 2018.
A) K-1 income from an S corporation. K-1 income from an S corporation is considered to be investment income. The other choices qualify as earned income for purposes of IRAs. Alimony resulting from divorce agreements prior to 2019 will count as earned income, but not after 2018 (2017 TCJA).
Which of the following plans is not permitted to offer a Roth account as part of the plan? A) Private 457(b) plan. B) Governmental 457(b) plan. C) 403(b) plan. D) 401(k) plan.
A) Private 457(b) plan. Private 457(b) plans cannot offer a Roth Account. The other plans are permitted.
Which of the following statements concerning a Sec. 423 employee stock-purchase plan is correct? Question 84 options: A) The effective discount can be substantially higher than 15 percent off the stock's price. B) The plan can be limited to a few highly compensated executives. C) Participants pay taxes at the time the stock is purchased. D) Like nonqualified stock options, the entire value of the discount is taxed as ordinary income.
A) The effective discount can be substantially higher than 15 percent off the stock's price. If the price of the stock rises over the purchase period and the price is 15 percent off the market price at the beginning of the period, the discount can vastly exceed 15 percent of the market price of the stock at the time of the purchase. Statement (B) is incorrect because these plans must cover most full-time employees. Statement (C) is incorrect because taxes are not paid until the stock is sold. Statement (D) is incorrect because only a portion of the value is taxed as ordinary income.
Which of the following statements regarding the plan sponsor of a money-purchase pension plan is correct? Question 22 options: A) The plan sponsor is required to make an annual contribution to the plan. B) The excess earnings of a money-purchase pension plan are returned to the plan sponsor. C) The plan sponsor generally bears the investment risk of the plan assets. D) A plan sponsor with fluctuating cash flows would adopt a money-purchase pension plan.
A) The plan sponsor is required to make an annual contribution to the plan. A money-purchase pension plan is a defined-contribution pension plan; therefore, it requires an annual contribution by the plan sponsor. Statements (B) and (C) are incorrect because the participant in a money-purchase pension plan generally bears all of the investment risk and benefit of the plan assets. Statement (D) is incorrect because an employer with fluctuating cash flows would not choose a money-purchase pension plan because of the mandatory funding requirement.
Roger and Robin were happily married until Roger fell in love with Sam. As a result, Roger and Robin have agreed they need to get a divorce. As part of the process, the court has provided a domestic relations order that calls for Robin's profit-sharing plan to be divided into equal portions such that Roger will have his own account with half of the value of the retirement account. What type of approach has been taken? Question 48 options: A) The separate interest approach B) The split payment approach C) The shared payment approach D) The divided account approach
A) The separate interest approach The separate interest approach calls for splitting a retirement account into two separate accounts. Each party is free to act with regard to his or her separate account without the interference or consent of the other party. There is no such term as a split payment approach or divided account approach.
Kevin established a Roth IRA and named his wife, Jennifer, as the sole beneficiary on the beneficiary designation form. Ten years later, he and Jennifer divorced. Kevin updated his will to leave all of his assets to his son, Brian. How will the IRA be distributed at Kevin's death? Question 65 options: A) To Jennifer B) To Brian C) To Brian and Jennifer equally D) To Kevin's estate - the executor will have to sort out the situation
A) To Jennifer The beneficiary form controls the distribution of IRA assets and not the will.
Joe Bob receives stock options (ISOs) with an exercise price of $18 when the stock is trading at $18. Joe Bob exercises these options two years after the date of the grant when the stock price is $39 per share. Which of the following statements is correct? Question 80 options: A) Upon exercise, Joe Bob will have no regular income for tax purposes. B) Joe Bob will have W-2 income of $21 per share upon exercise. C) Joe Bob will have $18 of AMT income upon exercise. D) Joe Bob's adjusted basis for regular income tax will be $39 at exercise.
A) Upon exercise, Joe Bob will have no regular income for tax purposes. Joe Bob does not have income at the date of exercise. Joe Bob's adjusted basis will be $18. The AMT income, if applicable, is equal to the difference between the fair market value and the exercise price ($39 − $18 =$ 21).
Abby works for See Corp. In January 2020, she was granted ten incentive stock options (ISOs) at See Corp's current value of $10 per share. In September 2022, Abby exercises all the ISO options when See Corp is worth $25 per share. In December 2024, Abby sells all her See Corp shares for $30 per share. All of the following statements are correct EXCEPT: Abby was not subject to ordinary income taxation when the ISOs were granted. Abby must pay ordinary income tax on the $150 gain between the grant price and the price at exercise ($25 − $10 = $15 gain per share). Abby must pay long-term capital-gain tax on the $20 gain per share between the grant date and sale date ($30 − $10 = $20 gain per share). Abby cannot gift her ISOs to her son.
Abby must pay ordinary income tax on the $150 gain between the grant price and the price at exercise ($25 − $10 = $15 gain per share). Abby will not incur ordinary income tax when she exercises the stock option. That is the treatment that applies to a nonqualified stock option (NQSO), not an ISO.
Which of the following types of retirement plans is allowed to borrow in order to purchase company stock? A stock bonus plan A profit-sharing plan An employee stock ownership plan (ESOP) A 401(k) plan
An employee stock ownership plan (ESOP) Stock bonus plans, profit-sharing plans, and 401(k) plans are incorrect because these plans do not allow the plan to borrow in order to purchase company stock. Only ESOP allows the plan to borrow in order to purchase company stock.
A distress termination of a qualified retirement plan occurs when which of the following occurs? The Pension Benefit Guaranty Corporation (PBGC) initiates a termination because the plan was determined to be unable to pay benefits from the plan. An employer is in financial difficulty and is unable to continue with the plan financially. Generally, this occurs when the company has filed for bankruptcy, either Chapter 7 liquidation or Chapter 11 reorganization. The employer has sufficient assets to pay all benefits vested at the time but is distressed about it. When the PBGC notifies the employer that it wishes to change the plan due to the increasing unfunded risk.
An employer is in financial difficulty and is unable to continue with the plan financially. Generally, this occurs when the company has filed for bankruptcy, either Chapter 7 liquidation or Chapter 11 reorganization The second option provides the definition of a distress termination. The third option describes a standard termination. The first option describes an involuntary termination and the fourth option is simply false.
Which of the following is an example of a benefit that will receive favorable tax treatment when offered through an employer's cafeteria plan? An employer providing the use of an on-premise gym for employees. An employer providing long-term disability coverage for employees. An employer providing tuition reimbursement for employees. An employer reimbursing employees for qualifying re-location expenses.
An employer providing long-term disability coverage for employees. Employers are able to provide both long-term and short-term disability coverage for employees under a cafeteria plan. All other answer options are examples of benefits that can be provided by employers but are unable to receive favorable tax treatment specifically under a cafeteria plan.
Which of the following statements regarding target-benefit pension plans is true? A target-benefit pension plan cannot allocate plan forfeitures to remaining plan participants' accounts. Target-benefit pension plans may not be established after 2001. Assuming equal salaries, a target-benefit pension plan would allocate a higher percentage of its current contributions to an older employee. A target-benefit pension plan may always exclude any participant who has not attained the age of 26 and completed one year of service.
Assuming equal salaries, a target-benefit pension plan would allocate a higher percentage of its current contributions to an older employee. A target-benefit pension plan uses a combination of age and compensation to determine the contribution to the plan on behalf of a participant. An older participant with a salary equal to the salary of a younger participant would receive a higher allocation for the year. Statement (A) is false because a target-benefit pension plan can allocate plan forfeitures to other plan participants' accounts. Statement (B) is false because target-benefit pension plans may be established after 2001. Statement (D) is false because a target-benefit pension plan must follow the general eligibility rules of 21 and 1.
Christian wants to retire in 15 years when he turns 65. Christian wants to have enough money to replace 75% of his current income less what he expects to receive from Social Security at the beginning of each year. He expects to receive $18,000 per year from Social Security in today's dollars. Christian is aggressive and wants to assume an 8% annual investment rate of return and that inflation will be 3% per year. Based on his family history, Christian expects that he will reach the age of 95. If Christian currently earns $80,000 per year and he expects his raises to equal the inflation rate, how much does he need at retirement to fulfill his retirement goals? Question 5 options: A) $1,022,807 B) $1,072,454 C) $1,559,131 D) $1,583,152
B) $1,072,454 Step 1: Estimate the funding amount in today's dollars ($80,000 x .75) = $60,000 - $18,000 = $42,000 Step 2: Inflate the need to the beginning of retirement. PV: <42,000>I/YR: 3N: 15PMT: 0FV: 65,434.63 Step 3: Determine the funding need at retirement age. BEGIN MODEPMT: 65,434.63I/YR: [(1.08/1.03) − 1] × 100 = 4.8544N: 30FV: 0PV: <1,072,454.29> Step 4: Determine the required annual savings amount. END MODEFV: 1,072,454.29N: 15I/YR: 8PV: 0PMT: <39,498>
Carolyn is planning for her retirement. She is currently 37 years old and plans to retire at age 62 and hopes to live until age 97. She currently earns $100,000 per year and anticipates needing 80% of her income during retirement. She anticipates that Social Security will provide her with $15,000 per year starting at age 62, leaving her with required savings to provide $65,000 annually during retirement ($100,000 × 0.80 − $15,000). She believes she can earn 11% on her investments, and that inflation will be 2% per year. How much must Carolyn save at the end of each year if she wants to make her last savings payment at age 62 to meet her retirement goal, assuming she wants to maintain the original purchasing power of her capital balance? Question 2 options: A) $4,759 B) $10,899 C) $11,464 D) $11,566
B) $10,899 Step 1: Determine the amount to be funded. 10BII Keystrokes Income today: $100,000 WRR: 80% Needs: $80,000 Less Social Security and Pension: $80,000 - $15,000 = $65,000 Step 2: Inflate funds to retirement age. PV: ($65,000) N: 25 I: 2.00% PMT: 0 Solve for FV: $106,639.39 Step 3: PV of retirement annuity due. PMT: $106,639.39 N: 35 I: [(1.11 / 1.02) - 1] x 100 = 8.8235% FV: 0 Solve for PV: ($1,247,035.59) Step 4: Annual funding amount. PDV: $1,247,035.59 N: 25 I: 11.00% PV: $0 Solve for PMT: ($10,899.37)
Monica, aged 40, receives $150,000 of group term life insurance from her employer as an employee benefit. The employer pays the premium equal to $300 for Monica. As a result, what will be Monica's taxable income? (Assume the cost of protection per $1,000 of protection for one month is $0.10 for employees ages 40 through 44). A) $0 (a non-taxable benefit) B) $120 (40% of the premium) C) $200 (2/3 of the premium) D) $300 (the full premium)
B) $120 (40% of the premium) $0.10 × 12 months = $1.20 per thousand × $100,000, (the excess over $50,000 group term) = $120.00.
ABC Inc. sponsors a defined contribution plan. Seth, age 39, has compensation of $150,000 for the year. ABC has made a $20,000 profit sharing plan contribution for Seth and $5,000 of plan forfeitures were allocated to Seth's profit sharing plan during the year. How much can Seth defer into his CODA plan (401(k) plan) for 2023? A) $12,500 B) $22,500 C) $30,000 D) $41,000
B) $22,500 The annual additions limit permits $66,000 (2023) of contributions to be made on Seth's behalf to qualified plans for the year. The employee elective deferral limit is $22,500 for 2023 and is included in the $66,000 calculation. In this question, Seth received a total contribution of $25,000 on his behalf into the plan so his deferral limit is not limited because of the $66,000 maximum. Seth can contribute the maximum employee deferral, or $22,500, to the plan for 2023. Answer choice a applies the 25% employer contribution limit to Seth's salary (25% x $150,000 = $37,500 - $25,000 profit sharing and forfeiture allocation = $12,500), which is incorrect. The employer's 25% contribution limit is based on the aggregate of all employees, not applied to each employee individually.
Milton, age 38, earns $170,000 per year. His employer, Dumaine Consulting, sponsors a qualified profit-sharing 401(k) plan and allocates all plan forfeitures to the remaining participants. If in the current year, Dumaine Consulting makes a 20% contribution to all employees and allocates $4,000 of forfeitures to Milton's profit-sharing plan account, what is the maximum Milton can defer to the 401(k) plan in 2023? Question 12 options: A) $0 B) $22,500 C) $23,000 D) $27,000
B) $22,500 The maximum annual addition to qualified plan accounts on behalf of Milton is $66,000 for the plan year. The maximum limit on annual additions is comprised of employer contributions, plan forfeiture allocations, and employee deferrals. If Dumaine contributes $34,000 ($170,000 x 20%) to the profit-sharing plan account and Milton receives $4,000 of forfeitures, there is $28,000 of contribution still allowable for Milton ($66,000 - $34,000 - $4,000= $28,000). However, he is limited to the $22,500 annual limit.
Susan's new employer is expecting to contribute $5,000 annually toward her pension account. She also has an option to include a whole-life insurance policy within that account. Given her current age, the level annual premiums would run $10 per $1,000 of death benefit. What is the maximum policy death benefit that her employer can purchase for Susan? Question 25 options: A) $125,000 B) $250,000 C) $375,000 D) $500,000
B) $250,000 Susan's pension account starts out empty but will gain $5,000 annually of employer contributions. According to the 25%/50% test, the aggregate of whole-life premiums can't exceed 50% of employer contributions, or $2,500 annually. Since this level premium policy costs $10 annually per $1,000 of death benefit, the maximum benefit is ($2,500 / $10) × $1,000 = $250,000.
Ronda turns 72 on February, 15th of year 2. She was a participant in her former employer's profit-sharing plan. Her profit-sharing plan has an account balance of $750,000 on December 31 of year 1, and $1,000,000 on December 31 of year 2. According to the Uniform Lifetime Table, the factors for ages 71 and 72 are 26.5 and 25.6 respectively. What will be the amount of Ronda's first required minimum distribution? Question 25 options: A) $28,302 B) $29,297 C) $37,736 D) $39,063
B) $29,297 $750,000/25.6 = $29,297. Use the previous year's balance and current factor knowing that Ronda reaches age 72 in Year 2. This will be the first year that she is required to take distributions whether in an IRA or a qualified plan.
Funny Dog, LLC, sponsors a defined-contribution plan. Kyle aged 58, has compensation equaling $100,000 for the year. Funny Dog has made a $30,000 profit-sharing plan contribution on Kyle's behalf, and $5,000 of plan forfeitures were allocated to Kyle's profit-sharing plan during the year. How much can Kyle defer into his CODA plan (401(k)) to maximize his annual contributions to the qualified plan for 2023? Question 9 options: A) $22,500 B) $30,000 C) $38,000 D) $34,000
B) $30,000 The annual additions limit permits $66,000 of contributions to be made on Kyle behalf to qualified plans for the 2023 tax year. Since he is over 50, we can add another $7,500 on top of that for a maximum of $73,500 or 100% of compensation. The employee elective deferral limit is $22,500 for 2023 and is included in the $66,000 calculation. In this problem, Kyle received a total contribution of $35,000 on his behalf into the plan, which still allows him to defer the maximum of $30,000 to the plan for 2023.
Don, single and aged 75, is a participant of his employer's qualified profit-sharing plan. He would like to make a deductible IRA contribution. If his adjusted gross income (AGI) is $55,000 (composed of $4,000 of W-2 earnings and $51,000 of Social Security and portfolio income), what is the maximum deductible IRA contribution Don can make for 2022? Question 31 options: A) $0 B) $4,000 C) $6,500 D) $7,500
B) $4,000 Even though he is over the age of 72, the SECURE Act now allows for a working individual to make a contribution to a traditional IRA. Given his earned income is limited to $4,000, this is the maximum he can contribute in this example.
Martin, aged 59, is a highly compensated employee and a 5% owner who is a participant in the company's Profit-Sharing Plan/401(k) plan. The company made a 20% profit-sharing plan contribution equaling $66,000 this year. Assuming the plan is a Safe Harbor Plan, what is the maximum amount that Martin can defer under the 401(k) during 2023? Question 17 options: A) $0 B) $7,500 C) $22,500 D) $30,000
B) $7,500 The Company contributed $66,000 (or $330,000 × 20%). This contribution maxes out Martin's ability to make contributions to qualified plans. However, since Martin is 50 or over, he can make catch-up contributions totaling $7,500 for 2023. The fact that Martin is highly compensated is irrelevant in this problem.
Jim, aged 51, made a $100,000 direct rollover from the after-tax portion of his workplace profit-sharing account into a Roth IRA. He had an adjusted basis in the after-tax account of $150,000. The fair market value of the account at the time of the rollover was $500,000. Calculate the taxable amount of the Roth conversion and any applicable penalty. Question 50 options: A) $100,000 taxable, $10,000 tax penalty B) $70,000 taxable, $0 tax penalty C) $70,000 taxable, $7,000 tax penalty D) $30,000 taxable, $0 tax penalty
B) $70,000 taxable, $0 tax penalty Direct rollovers are exempt from the section 72(t) early withdrawal penalty tax, so the penalty must be $0. To calculate the amount of the Roth conversion contribution that is tax-free, the adjusted basis in the plan is divided by the fair market value of the plan as of the day of the distribution. This ratio is then multiplied by the gross rollover amount. Thus $30,000 of the $100,000 distribution is the return of the adjusted taxable basis: ($150,000 / $500,000) × $100,000. Accordingly, $70,000 ($100,000 − $30,000) will be subject to income tax.
Profit-sharing plans and stock bonus plans have similarities and differences. Which of the following statements are correct regarding these plans? Both plans allow participants to vote their shares held by the plan. Stock bonus plans and profit-sharing plans can make lump-sum distributions of employer securities at the termination of an employee. A) 1 only. B) 2 only. C) Both 1 and 2. D) Neither 1 nor 2.
B) 2 only. Only participants in a stock bonus plan can vote their shares. Both plans can distribute employer stock.
Which of the following statements is correct regarding the calculation of self-employment tax? 1. For a taxpayer who has both W-2 income and self-employment income, W-2 income earned during a year will always reduce the amount of self-employment tax due on self-employment income. 2. Two separate sources of self-employment income for a taxpayer are combined to determine the total self-employment tax owed by the taxpayer. A) 1 only. B) 2 only. C) Both 1 and 2. D) Neither 1 nor 2.
B) 2 only. Statement 1 is correct if the total of W-2 income and self-employment income exceeds the annual wage base. It is not true if it does not exceed the wage base. Statement 2 is correct.
Which of the following vesting schedules may a top-heavy qualified cash balance plan use? A) 2-to-6-year graduated. B) 3-year cliff. C) 3-to-7-year graduated. D) 5-year cliff.
B) 3-year cliff. Cash balance plans must vest at least as fast as a three-year cliff vesting schedule. They cannot use the 2-to-6-year graduated schedule
Which of the following vesting schedules may a top-heavy qualified-cash-balance plan use? Question 13 options: A) 1- to 4-year graduated B) 35% after 1 year, 70% after 2 years, and 100% after 3 years C) 2- to 6-year graduated D) 4-year cliff
B) 35% after 1 year, 70% after 2 years, and 100% after 3 years As a result of the Pension Protection Act of 2006, cash-balance plans must vest at least as fast as a three-year cliff-vesting schedule. The only choice that is possible is option (B).
Which of the following is an example of a qualified retirement plan? Question 15 options: A) Rabbi trust B) 401(k) plan C) Nonqualified stock option plan D) ESPP
B) 401(k) plan A 401(k) plan is a qualified plan. All of the others are not qualified retirement plans.
What is the earliest age that an IRA catch-up contribution can be made? A) 45 B) 50 C) 60 D) 65
B) 50 Catch-up contributions are permitted for those who have attained the age of 50.
Tidewater Company has 1,000 eligible employees and sponsors a defined benefit pension plan. The company is unsure if they are meeting all of their testing requirements. How many employees (the minimum) must be covered by Tidewater's defined benefit pension plan for the plan to conform with ERISA? A) 40 B) 50 C) 400 D) 500
B) 50 The 50/40 rule requires that defined-benefit plans cover the lesser of 50 employees or 40 percent of all eligible employees. In this example, 50 employees is less than 40 percent of 1,000, or 400 employees.
John would like to offer his 35 employees some form of retirement plan. His concerns in offering a plan include cost, administrative complexity, and mandatory funding requirements. John would like to be able to have a vesting period as long as 6 years, the ability to have flexibility in funding, and have employee deferrals available in the plan. Which of the following plans should John consider to best address his concerns? Question 19 options: A) A Money Purchase plan B) A Stock Bonus plan C) A Cash Balance plan D) A 401(k) plan
B) A Stock Bonus plan Answer choices (A), (C), and (D) have higher costs, greater administrative expenses and less funding flexibility than a Stock Bonus plan.
A cross-purchase buy-sell agreement is most appropriate for which of the following organizations? A) A sole proprietorship B) A partnership with 3 partners C) A partnership with 18 partners D) A large C-corporation
B) A partnership with 3 partners A cross-purchase buy-sell agreement is most appropriate for a small partnership. For larger organizations, the number of individual life insurance policies required to create a cross-purchase buy-sell agreement makes them too complex and cost-prohibitive. In those cases, an entity-purchase buy-sell agreement would be more appropriate.
All of the following are characteristics of pension plans EXCEPT: Question 11 options: A) They can only hold 10% of employer securities in the plan. B) All pension plans can skew benefits to longer serving employees. C) Pension plans require a contribution each plan year. D) They are mostly funded by employer contributions.
B) All pension plans can skew benefits to longer serving employees. All of the answers are correct statements except (B). Only defined benefit pension plans can account for prior service at plan inception.
Individual accounts in 403(b) plans may not be in which one of the following forms? Question 75 options: A) A contract provided through an insurance company called an annuity contract. B) An account invested in individual stocks in companies in the S&P 500. C) An account for church employees that is a retirement income account, which invests in either annuities or mutual funds. D) An account invested in mutual funds only, referred to as a custodial account.
B) An account invested in individual stocks in companies in the S&P 500 Individual accounts in 403(b) plans must be options (A), (C), or (D) and thus investing in individual stocks in S&P 500 companies is not allowed. Such stock may be owned through mutual funds only.
Which of the followings statements regarding 403(b) plans is true? Question 72 options: A) Assets within a 403(b) plan may be invested in individual stocks. B) Assets within a 403(b) plan may be invested in mutual funds. C) All 403(b) plans must pass the ADP test. D) In certain situations, a participant of a 403(b) plan can defer an additional $25,000 (catch-up) to a 403(b) plan in a single plan year.
B) Assets within a 403(b) plan may be invested in mutual funds. Only a contributory 403(b) plan has to pass the ADP test. 403(b) plans are not required to meet the ADP test, but 403(b) plans with employer contributions are required to meet the ACP test. The catch-up amount is $15,000, not $25,000.
All of the following are examples of fringe benefits that may be non-taxable to the employee EXCEPT: Question 43 options: A) Meals furnished for the convenience of the employer B) Athletic facility membership near the workplace paid for by the employer C) Dependent care assistance programs D) Educational assistance programs
B) Athletic facility membership near the workplace paid for by the employer Answer choices (A), (C), and (D) are all fringe benefits that may be offered by the employer and not deemed to be compensation to the employee. Answer choice (B) would not be deemed to be a non-taxable fringe benefit unless the athletic facilities are (1) operated by the employer, (2) located on the employer's premises, and (2) and "substantially all" of the use of the facility is by the employees of the employer.
Which of the following qualified plan distributions will be subjected to a 10% early withdrawal penalty in 2023? Question 24 options: A) Oliver, aged 56, was laid off by his employer. He takes a $5,000 distribution from his retirement plan to pay for living expenses. B) Bob, aged 45, takes a $100,000 distribution from his profit-sharing plan to pay for a luxury sports car. C) Iggy, aged 41, takes a $750,000 distribution from his employer's profit-sharing plan. Six weeks after receiving the check, he rolled over the funds into a new IRA account. D) Sam, aged 55, begins taking equal distributions from her qualified plan over her life expectancy. The annual distribution is $5,000.
B) Bob, aged 45, takes a $100,000 distribution from his profit-sharing plan to pay for a luxury sports car. Bob's distribution will be subjected to the 10% penalty. A distribution for a luxury sports car would most likely not be seen as a hardship. All the other statements are exceptions to the 10% early withdrawal penalty. Statement (A) describes the exception for separation from service after age 55. Statement (C) describes the exception for the rollover of qualified plan assets. Statement (D) describes the exception for substantially equal periodic payments.
Which of the following generally contribute(s) to defined-benefit plans, profit-sharing plans, and money-purchase pension plans? Question 52 options: A) Employees only B) Employer only C) Both employer and employees D) Employer, employees, and government
B) Employer only Employees contribute to 401(k) plans and thrift plans. The government does not contribute to plans, except when it is the employer.
If an employee earning $60,000 had access to multiple plans, which combination would allow for the largest employee contributions? A) 401(k) plan and profit sharing plan. B) Governmental 457(b) plan and a 403(b) plan. C) Profit sharing plan and SEP. D) 401(k) plan and a 403(b) plan
B) Governmental 457(b) plan and a 403(b) plan. The employee could defer $22,500 (2023) to the 457 plan and the 403(b) plan for a total of $45,000 in 2023. That amount is larger than the other combinations. The reason for this is that the "one deferral per year rule" that applies to 401(k) plans and 403(b) plans does not apply to 457 plans.
Dr. Singh is a participant in a stock bonus plan. The value of the employer stock contributions to the plan over the course of his participation totaled $100,000. On December 1st of last year, Dr. Singh decided to execute an NUA strategy and took a full distribution of the employer stock from the plan at a value of $500,000. This year, Dr. Singh sells all the stock for $400,000. Which of the following statements is true? Question 21 options: A) He has a long-term capital gain of $400,000 for last year. B) He has ordinary income of $100,000 for last year. C) He has a long-term capital loss of $100,000 in the current year. D) He has ordinary income of $100,000 and long-term capital gain of $400,000 for last year.
B) He has ordinary income of $100,000 for last year. Because Dr. Singh is taking a lump-sum distribution from a qualified plan of employer stock, he will not have to recognize the net unrealized appreciation until he disposes of the employer stock. However, at the time of the distribution, the value of the stock, as of the date of contribution to the plan, will be taxable as ordinary income. Any gain on the subsequent sale of the stock will be taxable as long-term capital gain. In this case, He will recognize $100,000 of ordinary income at the date of the distribution and long-term capital gain of $300,000 (that is, $400,000 − $100,000) at the date of sale.
Randy, aged 63, is a participant in a stock-bonus plan sponsored by XYZ, Inc., a closely held corporation. Over the years, Randy's account was credited with $15,875 of share purchases by XYZ. Randy terminates employment in August of this year and takes a distribution of the whole plan, which consists of 975 shares of XYZ, Inc. with a fair market value of $24,000. Randy plans to take advantage of Net Unrealized Appreciation (NUA). If Randy sells the stock for $40,000 next year, one year and one month after receiving the distribution, which of the following statements is (are) true? I. Randy has ordinary income of $15,875 when he executes the NUA. II. Randy has long-term capital gain of $24,000 when he executes the NUA. III. Randy has short-term capital gain of $8,125 when he executes the NUA. IV. Randy has long-term capital gain of $24,125 at the sale of the stock. Question 35 options: A) I only B) I and IV C) II and
B) I and IV Randy's ordinary income is exactly equal to XYZ, Inc.'s deduction at the time of contribution, $15,875, and this will be taxable as ordinary income in the year of the distribution from the Plan. Randy's net unrealized appreciation, $8,125 ($24,000 − $15,875), will be taxed as a long-term capital gain at the date of sale. Since Randy sold the stock at $40,000 one year and one month after taking the distribution, he will have $16,000 ($40,000 − $24,000) in additional long-term capital gains next year on the appreciation after distribution. His total long-term capital gains due at sale of the stock would be $8,125 + $16,000 = 24,125.
Which of the employee fringe benefits listed below, if provided by the employer, are both deductible by the employer and not included in an employee's gross income after 2017? I. Business periodical subscriptionsII. Season tickets to professional football gamesIII. Parking provided near its business (employer pays $90 per month)IV. The use of an on-premises athletic facility (value of $180 per employee per month) Question 89 options: A) II only B) I and IV C) I, III, and IV D) I, II, III, and IV
B) I and IV Season tickets to professional football games are includible in the gross income of the employee receiving the tickets. Periodicals and athletic facilities are both deductible and not included in gross income. Effective with TCJA 2017, season tickets to professional football games are deductible by the employer as compensation expense as long as the cost is includible in the gross income of the employee receiving the tickets. Qualified transportation fringe benefits are not deductible by the employer, as a result of the 2017 Tax Cuts and Jobs Act.
Which of the following is a requirement for eligibility to participate in a SIMPLE IRA plan? I. The participant must be over the age of 21II. The participant must have over $5,000 of earnings in the previous two years Question 35 options: A) I only B) II only C) Both I and II D) Neither I nor II
B) II only Statement I is false because no age restrictions are imposed for SIMPLE IRA plans.
Which of the following statements is true? I. All Pension Plans are required to participate in Pension Benefit Guarantee Corporation (PBGC)II. Cash Balance Plans with 50 Participants must participate in the PBGC Question 24 options: A) I only B) II only C) Both I and II D) Neither I nor II
B) II only Statement l is incorrect as not all pension plans require participation in the PBGC. Only defined benefit plans require participation unless they cover professional service corporations with 25 or fewer participants.
Cafeteria plans allow employees to receive cash or defer the receipt of cash to purchase certain qualified tax-free fringe benefits. Which of the following statements about cafeteria plans is true? I. Cafeteria plans can offer Group Term Life Insurance policies in excess of $50,000, of which the cost of the policy would not be included in the employees' gross income.II. Cafeteria plans can offer Group Term Life Insurance policies of less than $50,000, of which the cost of the policy would not be included in the employees' gross income. Question 48 options: A) I only B) II only C) Both I and II D) Neither I nor II
B) II only While cafeteria plans can offer group term life insurance policies in excess of $50,000, premiums in excess of the $50,000 limit for these policies would be included in the employees' gross income.
Non-qualified deferred compensation plans often use life insurance for funding within the plan. Why is this? A) Life insurance investment products typically have higher returns. B) Life insurance products have tax advantages. C) Life insurance provides an additional layer of avoidance for constructive receipt. D) Life insurance limits the liability of the company sponsor for the NQDC plan.
B) Life insurance products have tax advantages. The earnings within the life insurance product are not taxed at the time they are earned. In contrast, earnings on deferred funds are taxed to the corporate sponsor (unless it is a secular trust).
Mike was awarded 1,000 shares of restricted stock in B Corp at a time when the stock price was $14. Assume Mike properly makes an 83(b) election at the date of the award. The stock vests 2 years later at a price of $12 and Mike sells it then. What are Mike's tax consequences in the year of sale? Question 81 options: A) Mike has W-2 income of $12,000. B) Mike has a long-term capital loss of $2,000. C) Mike has W-2 income of $14,000. D) Mike has a $12,000 long-term capital gain.
B) Mike has a long-term capital loss of $2,000. In the year of sale, Mike will have a long-term capital loss of $2,000 ($14,000 − $12,000) because his right to the stock was vested. When 83(b) is elected, losses are permitted after the right to the stock has vested.
James retires after 50 years of working for Zacker Industries. His boss gives him a $400 Apple watch as a retirement gift. Is it taxable to James? A) No. Retirement gifts are not taxable if they are under $1,600. B) No. It does not exceed the $400 limit. C) Yes. All gifts from employers are considered compensation. D) Yes. Gifts given without a qualified award plan are taxable.
B) No. It does not exceed the $400 limit. The value of the gift is relatively small ($400 or smaller) and, thus, not included. The TCJA 2017 requires the gift to be tangible personal property. The Apple watch certainly qualifies as tangible personal property.
Which the following people may make a deductible contribution to a traditional IRA for 2023? Person -Adjusted Gross Income (AGI)-Active Participant of Retirement - Plan - MaritalStatus A.Debby-$1,000,000-Yes-Married B.Pam-$500,000-No-Single C.Craig-$95,000-Yes-Single D.James-$0-No-Single Question 32 options: A) Debby B) Pam C) Craig D) James
B) Pam Only Pam may make a deductible contribution to a traditional IRA. Craig's AGI is above the 2023 phaseout for single tax filers ($73,000 to $83,000). Debby's AGI is above the 2023 phaseout for MFJ and she is an active participant in a retirement plan, while James does not have any earned income.
Paris was awarded 1,000 shares of restricted stock of Diamond Corporation at a time when the stock price was $7. Assume Paris properly makes an 83(b) election at the date of the award. The stock vests three years later at a price of $19 and Paris sells it then. What are Paris's tax consequences in the yearshe sells the stock? A) Paris has W-2 income of $19,000. B) Paris has a long-term capital gain of $12,000. C) Paris has W-2 income of $12,000. D) Paris has a $19,000 long-term capital gain.
B) Paris has a long-term capital gain of $12,000. At the time Paris makes the 83(b) election, the value of the stock at that date will be included in her taxable income. Thus, Paris will have W-2 income of $7,000 ($7 x $1,000) in the year the stock was awarded. Any gain beyond that will be capital. Thus, $12,000 will be capital gain.
All of the following are true regarding employer contributions to secular trusts for employee-participants of a nonqualified deferred-compensation agreement EXCEPT: Question 79 options: A) Participants have security against an employer's change of heart. B) Participants have no security against an employer's bankruptcy. C) Secular trusts may provide tax deferral for employees until vesting. D) Secular trusts may provide employers with a current income tax deduction.
B) Participants have no security against an employer's bankruptcy. Secular trusts are similar to rabbi trusts except that participants do not have a substantial risk of forfeiture and thus may not provide the employee with tax deferral. Depending on the type of Secular trust (employee and employer) , the trust may provide the employer with a current income tax deduction for trust contributions. Secular trusts protect the participant from employer unwillingness to pay because they are funded. They also may also protect from bankruptcy, because the assets are located in trust and are not the asset of the business. Whether the plan falls under Title I of ERISA will establish the degree of bankruptcy protection it has.
Mary is interested in installing a qualified plan for her employees and herself in her travel agency. She likes the idea of allowing herself and her employees maximum flexibility with regards to being able to access plan assets without incurring heavy taxes or penalties in case of need or hardship. Given her preferences, which qualified plan option is most appropriate? Question 6 options: A) Money Purchase Pension Plan B) Profit-Sharing Plan/401(k) Plan C) Cash Balance Plan D) 403(b) Plan
B) Profit-Sharing Plan/401(k) Plan Mary is interested in installing a qualified plan so that immediately erases answer choice (D) as an option, since 403(b)'s are not considered qualified plans. Choices (A) and (C) are pension type plans that limit withdrawals and do not normally provide loan options for participants.
All of the following are permitted rollovers EXCEPT: Question 45 options: A) 401(k) to Roth 401(k) B) Roth 401(k) to Traditional IRA C) 403(b) plan to a qualified plan D) Qualified plan to a governmental 457(b) plan
B) Roth 401(k) to Traditional IRA Roth 401(k) plans can be rolled over only to another similar plan or to a Roth IRA; rollovers into traditional IRAs are not permitted. All of the other rollovers are permitted.
Cindy Sue has been with CS Designs, Inc. for five years. CS Designs has a deferred compensation plan to provide benefits to key executives only. CS Designs contributed $400,000 into a trust for Cindy Sue's benefit under the company's deferred compensation plan. The plan requires that executives must work for the company for 10 years before any benefits can be obtained from the plan. Cindy Sue has come to you to determine when she will be subject to income tax on the contribution by the employer. Which of the following is correct? Question 83 options: A) Since the assets were placed into a trust, the economic benefit doctrine will require inclusion in income for the current-year contributions made by the employer. B) Since Cindy Sue cannot receive the benefits until she has been with the employer for 10 years, the substantial risk of forfeiture doctrine will not require inclusion in income for the current-year contrib
B) Since Cindy Sue cannot receive the benefits until she has been with the employer for 10 years, the substantial risk of forfeiture doctrine will not require inclusion in income for the current-year contributions made by the employer. The economic benefit and constructive receipt doctrines will not cause inclusion because the assets are forfeitable if she does not stay with the company for the required length of service. Deferred compensation plans are by nature discriminatory. The contributions will be included if the employee has an economic benefit, no risk of forfeiture, or constructive receipt.
A SEP is not a qualified plan and is not subject to all of the qualified plan rules. However, it is subject to many of the same rules. Which of the following is a true statement? Question 67 options: A) SEPs and qualified plans have different funding deadlines. B) The contribution limit for SEPs and qualified plans (defined contribution) is $66,000 per employee for the year 2023. C) SEPs and qualified plans have the same ERISA protection from creditors. D) SEPs and qualified plans have different nondiscriminatory and top-heavy rules.
B) The contribution limit for SEPs and qualified plans (defined contribution) is $66,000 per employee for the year 2023. SEPs and qualified plans can be funded as late as the due date of the return plus extensions. The maximum contribution for an individual to a SEP is $66,000 for 2023. Thus statement B is correct. Qualified plans are protected under ERISA. IRAs and SEPs do not share this protection, though there are protections for IRAs under the Bankruptcy Abuse Prevention and Consumer Protection Act (BAPCPA) of up to 1MM for IRAs. Assets rolled over from a qualified plan retain their ERISA bankruptcy protection as fruits of a qualified plan. Both types of plans have the same nondiscriminatory and top-heavy rules.
Sew What, the best seamstress shop in town, sponsors a 401(k) plan. The plan provides a dollar-for-dollar match for employee contributions up to six percent and has immediate vesting for all contributions. For actual deferral percentage (ADP) purposes, the company has not made the top 20 percent election for the determination of who is highly compensated. The company has the following employee information: Employee-Ownership-Compensation-Elective Deferral-Deferral Percent Lois - 93% - $201,000- $14,000 -6.97% Frank - 5% - $150,000 - $14,000 - 9.33% Karen - 2% - $140,000 - $12,600 - 9.00% Jeanette ─ - $40,000 - $4,000 - 10.00% Joyce - − - $30,000 - ─ - 0.00% Ronnie - ─ - $30,000 - $1,800 - 6.00% Kali - − - $30,000 - − - 0.00% A) Karen is not highly compensated. B) The plan passes the ADP test if Joyce and Kali were not eligible. C) Joyce and Kali are not considered when calculating the ADP test because t
B) The plan passes the ADP test if Joyce and Kali were not eligible. Lois, Frank and Karen are highly compensated employees. Lois is the only one who meets the ownership test of greater than five percent. Lois, Frank, and Karen meet the earnings test for being highly compensated. The plan fails the ADP test. The NHC average ADP is 4% [(10% + 0% + 6% + 0%) / 4] while the HC ADP is 8.43% [(6.97% + 9.33% + 9.00%) / 3]. Thus using the test, 2 is added to the NHC ADP of 4% requiring that the HC ADP be at or below 6%. Since the HC ADP is 8.43%, the plan fails the ADP test. If Joyce and Kali were not eligible, then the NHC ADP equals 8% and the plan meets the ADP test. Options (C) and (D) are not correct, since there is no indication that the plan is top heavy. In addition, top heavy applies to key employees, not highly compensated employees.
Which of the following expenditures will most likely increase during retirement? Question 7 options: A) Clothing costs B) Travel C) FICA D) Savings
B) Travel Travel is the most likely expenditure to increase during retirement. Many other costs will likely be reduced after the retiree leaves the workforce, including a reduction in clothing expenses and the elimination of payroll taxes. An individual's savings may also be eliminated because a retirement plan requires the use of the accumulated savings during retirement.
Which of the following statements concerning cafeteria employee-benefit plans is correct? Question 96 options: A) Cafeteria plans are the answer to the real world, where family benefit needs are homogeneous. B) Under a cafeteria plan, a family can write its own benefit plan by selecting among options available. C) The employer may offer educational assistance as a benefit under a cafeteria plan. D) Most cafeteria plans offer a pension plan or some type of deferred-compensation plan.
B) Under a cafeteria plan, a family can write its own benefit plan by selecting among options available. Statement (A) is not correct. Cafeteria plans are suited for groups in which family benefit needs are NOT homogeneous. Statement (C) is not a correct statement. Educational assistance is not an eligible benefit under a cafeteria plan. Statement (D) is not correct. 401(k) plans are the only type of retirement benefit plans that may be offered through a cafeteria plan.
This is the type of account that permits employees to buy additional employee benefits using pre-tax dollars. A) pre-tax plan B) flexible spending account C) contributory plan D) managed plan
B) flexible spending account
All of the following are characteristics of pension plans EXCEPT Question 19 options: A) mandatory funding. B) in-service withdrawals for employees under the age of 59 ½ years. C) limited investment in life insurance. D) the limit of a 10 percent investment in the employer's securities.
B) in-service withdrawals for employees under the age of 59 ½ years. A pension plan requires mandatory funding, limits the investment of plan assets in life insurance, and limits the investment of plan assets in the employer's securities. A pension plan may not allow in-service withdrawals for employees under the age of 59 ½ years.
Carla would like to determine her financial needs during retirement. All of the following are costs she might eliminate in her retirement needs calculation EXCEPT Question 1 options: A) the $175 per month of parking expenses for parking at her place of employment. B) the $1,500 mortgage payment that is scheduled to end 12 years into retirement. C) the Medicare taxes she pays each year. D) the $2,000 per month she deposits into savings.,
B) the $1,500 mortgage payment that is scheduled to end 12 years into retirement Carla would not eliminate her mortgage since it will not be paid off at retirement. She would eliminate the parking expense, Medicare taxes, and savings expense since she would most likely no longer have these expenses during retirement.
Which of the following are acceptable reasons for an employer to terminate a qualified retirement plan? 1. The employer is not profitable and cannot afford to make plan contributions. 2. The employer wants to reduce the cost of retirement benefits. As a result, the employer terminates a defined benefit plan and replaces it with a 401(k) plan. A) 1 only. B) 2 only. C) Both 1 and 2. D) Neither 1 nor 2.
Both 1 and 2.
Defined-benefit plans and cash-balance plans are both pension plans. However, they are significantly different types of plans. Which of the following statements is true? Question 4 options: A) A cash-balance pension plan benefits younger and older employees equally. B) Both plans have hypothetical accounts for each plan participant. C) Both plans use the same vesting schedules regardless of whether the plan is top heavy. D) Both plans can provide for lump-sum benefits upon termination and/or full retirement age.
Both plans can provide for lump-sum benefits upon termination and/or full retirement age. Statement (A) is false because cash-balance pension plans usually benefit younger participants because of the guaranteed contribution rate and the guaranteed earnings rate. Statement (B) is false because a defined-benefit plan promises a defined benefit at the participant's retirement. It does not have a hypothetical account like a cash-balance plan. Statement (C) is false because cash-balance plans must use the 3-year cliff vesting.
Harvey and Morgan were just married, but Harvey does not work. They are both 25 years old. Morgan supports Harvey with funds from her trust fund, investment income of $12,000, and her part-time earnings of $7,500 this year. If Morgan contributes $6,500 in 2023 to her Roth IRA this year, how much can Harvey contribute to his traditional IRA? A) $0 B) $250 C) $1,000 D) $6,500
C) $1,000 Harvey and Morgan have combined earned income of $7,500 for the current year, which limits his ability to contribute to his IRA. $7,500 − $6,500 = $1,000.
Mike, aged 39, earns $100,000 annually from Royalty, Inc. Royalty sponsors a SIMPLE and provides a dollar-for-dollar match of all employee deferrals up to a 3% contribution. What is the maximum overall contribution that may be made to his SIMPLE account for 2023? Question 38 options: A) $6,000 B) $14,000 C) $15,500 D) $22,500
C) $15,500 He can defer up to $15,500, the maximum SIMPLE deferral for 2023. Royalty can match up to 3% of his compensation, or $3,000 (that is, 3% × $100,000). The maximum contribution to his SIMPLE is therefore $18,500 (or $15,500 + $3,000).
Marcus left his Company last July after completing 4 years of service. When at Stronger, Marcus participated in the Profit-Sharing Plan/401(k) Plan. The company provides a 100% match up to 3%. Marcus had a total account value of $200,000 in the Plan, of which. $100,000 represented a direct transfer from his previous employer and $40,000 was attributable to non-elective contributions that Stronger made for all non-excludable employees. The remaining $60,000 was a combination of Marcus' salary contributions from a CODA he set up upon completing 1 year of service with Stronger and the company match of those contributions. Assume that the deferral and match amounts are equal. At this time, considering Marcus has terminated employment and the Plan followed the least generous graduated vesting schedule, what is Marcus' vested account balance in the 401(k) profit-sharing plan? A) $100,000 B) $130,000 C) $172,000 D) $200,000
C) $172,000 Marcus would be immediately vested in his contributory amounts to the Plan which would be the $100,000 he transferred into the Plan as well as the $30,000 that he contributed as part of the Salary Reduction or CODA (401(k)). Along with this, using a 2-to-6 year graduated vesting schedule, after 4 years Marcus would be 60% vested in the nonelective contributions of $40,000 and the matching contributions of $30,000 or he would be vested in $42,000 in noncontributory contributions. Adding $130,000 of vested contributory amounts and $42,000 of non-contributory amounts allows us to arrive at $172,000.
Jodie is covered by a $500,000 group term life insurance policy of which his wife is the sole beneficiary. His employer pays the entire premium for the policy, for which the uniform annual premium is $0.43 per $1,000 per month of coverage. How much, if any, of the cost of the group term life insurance is included in his gross income assuming 12 months of coverage is provided? Question 49 options: A) $0 B) $194 C) $2,322 D) $2,580
C) $2,322 Jodie can exclude the cost of up to $50,000 of group term life insurance coverage. In this case, the cost of $50,000 of coverage is $258, that is, [($50,000 ÷ $1,000) × 0.43] × 12 months. Therefore, the remaining $2,322 cost of coverage must be included in his gross income, [($500,000 − $50,000) ÷ $1,000] × 0.43 × 12 months
Harper, age 45, earns $400,000 annually from Atlas, which sponsors a SIMPLE, and makes the minimum required non-elective contribution to all eligible employees. What is the maximum total contribution to Harper's account in 2023, including both employee and employer contributions? A) $15,500 B) $19,000 C) $22,100 D) $23,500
C) $22,100 Harper can defer up to $15,500 for 2023. The non-elective contribution equals 2% of his salary limited to the 2023 limit of $330,000, which is $6,600. Thus, the total equals $22,100.
Jon, age 39, is a highly compensated employee who earns $175,000 per year and is a participant in his employer's 401(k). His employer made a 20% profit-sharing plan contribution during the year equaling $35,000. Ignoring the actual deferral percentage (ADP) test requirements, what is the maximum amount that Jon can contribute to the 401(k) via a pre-tax salary deferral for 2023? Question 29 options: A) $0 B) $6,500 C) $22,500 D) $30,000
C) $22,500 The total contribution limit per participant in 2023 is $66,000. Jon's employer contributed $35,000 (or $175,000 × 20 percent), leaving $66,000 - $35,000 = $31,000 of possible additional contributions into his 401(k). However, the maximum salary deferral permitted in 2023 is $22,500. Therefore, Jon can only contribute the maximum 401(k) contribution amount of $22,500 in 2023 via a pre-tax salary deferral. The fact that Jon is highly compensated is irrelevant in this problem.
Acme Corporation sponsors a profit-sharing plan. Acme wants to use a profit-sharing allocation method that allocates profits based on the percentage of the employee's respective salaries to all participants. If $200,000 is contributed as a profit share by Acme, how much will employee C receive? Employee -Age - Compensation A - 55 - $250,000 B - 45 - $60,000 C - 35 - $50,000 D - 30 - $40,000 $400,000 Question 15 options: A) $10,000 B) $20,000 C) $25,000 D) $30,000
C) $25,000 Using a method that ensures that all employees receive the same percentage of their respective salaries, employee C will receive $25,000. A - 55 - 250,000 - 63% - 125,000 B - 45 - 60,000 - 15% - 30,000 C - 35 - 50,000 - 13% - 25,000 D - 30 - 40,000 - 10% - 20,000 400,000 - 100% - 200,000
Soraya wants to retire in 30 years when she turns 60. She wants to have enough money to replace 80% of her current income. She expects to receive $40,000 per year from Social Security in today's dollars at 60 (assuming this is permissible when she retires). Soraya wants to assume an 8% annual investment rate of return while also assuming that inflation will be 4% per year. Soraya wants to plan to live until age 110 to ensure that she doesn't run out of money. If she currently earns $105,000 per year and she expects her raises to equal the inflation rate, approximately how much does she need at retirement to fulfill her retirement goals (rounding is assumed)? Question 4 options: A) $3,056,292.65 B) $3,132,739.45 C) $3,269,288.79 D) $3,272,305.96
C) $3,269,288.79 The answer is calculated by completing the following steps. Answers may range due to rounding. Step 1: Use the following figures to determine the future value of the money that Susan will need to replace: PV = $44,000 = (0.80 × $105,000) − $40,000 N = 30 i = 4% PMT = $0 The future value is $142,709.49. Step 2: Determine the present value of that amount: BEGIN MODE FV = 0 N = 50 i = 3.8462% = [(1.08 ÷ 1.04) − 1] × 100 PMTAD = $142,709.49 The present value of that amount is $3,269,288.79.
Jacques, who is 52 years old, works for NYU and is a participant in NYU's tax-sheltered annuity (TSA) program. How much could he defer in the TSA in 2023 if he has a salary of $121,000? A) $7,500. B) $22,500. C) $30,000. D) $66,000.
C) $30,000 The deferral for a 403(b) plan or TSA is $22,500 for 2023. There is an additional catch-up contribution of $7,500 that is allowed for individuals who have attained age 50.
Larry, age 55, is employed by BB Trucking Company as a tire repair specialist. He earns $65,000 per year and received an allocation of $40,000 to his employer-provided profit-sharing plan for the year. If BB Trucking does not match employee deferrals, what is the maximum amount Larry can defer to his 401(k) plan for the 2023 plan year? Question 28 options: A) $16,000 B) $22,500 C) $30,000 D) $32,000
C) $30,000 The maximum deferral to a 401(k) plan for a participant who is over 50 years old in 2023 is $30,000 ($22,500 plus catch-up of $7,500). Since Larry has received an allocation from the profit-sharing plan of $40,000, he is able to defer $22,500 (<$66,000 − $40,000) plus the $7,500 catch-up deferral for participants who are 50 years old and older to maximize his defined-contribution plan limit for the year.
Barbie, aged 45, is a highly compensated employee of Tarjay, Inc., and she participates in their nonqualified deferred-compensation program. The year prior to this, she elects to have 10% of her $300,000 annual salary (or $30,000) deferred into the nonqualified plan, which is held in a secular trust with no vesting requirement. Her distribution election is to receive her deferred compensation when she turns 50 years old. Her contribution will be informally funded in a mutual fund account that is expected to increase to $50,000 at age 50. Based on this information, how much of Barbie's compensation will be subject to ordinary income rates in this year? Question 42 options: A) $30,000 B) $270,000 C) $300,000 D) $250,000
C) $300,000 Because a secular trust with no vesting schedule is being used, there is no substantial risk of forfeiture. Therefore, none of the $30,000 contribution would be deferred for income tax purposes this year. Instead, the entire $300,000 of compensation would be subject to ordinary income tax rates this year.
On January 15, Year 1, Buddy, a Senior Vice President for Creative Cake Corporation (CCC), is granted 20,000 ISOs at an exercise price of $10. On February 6, Year 2, he exercises all his options when the price of CCC stock is $27. He sells the ISO shares three years later, when CCC stock is trading at $47 per share. What is his long-term capital gain per share? A) $20. B) $27. C) $37. D) $47.
C) $37. His basis of the ISO shares (for regular tax) is $10. Thus, the difference between $47 and $10 is the capital gain.
James, aged 58, has compensation of $150,000 and wants to defer the maximum to his public 457(b) plan. The normal retirement age for his plan is age 60. How much can he defer in 2023 if he has an unused deferral amount of $60,000 from age 40 to age 49? Question 73 options: A) $20,500 B) $25,000 C) $45,000 D) $52,000
C) $45,000 He can contribute $45,000 (2 × $25,500). He must be within three years of retirement and have unused deferral amounts. Note that since he used the final 3-year catch-up, he cannot use the age-50-and-over catch-up.
Jordan contributed $5,000 each year to her Roth IRA for eleven years. At age 57, Jordan's IRA was worth $100,000 consisting of $55,000 in contributions, $25,000 in conversions from her 401(k) plan last year, and earnings of $20,000. What are the tax consequences if Jordan takes a complete distribution of the Roth IRA at age 57, once she has retired, to travel around the world? Question 66 options: A) Only $20,000 is subject to the 10 percent early withdrawal penalty. B) $20,000 is subject to income tax but no penalty. C) $45,000 is subject to the 10 percent early withdrawal penalty and $20,000 is subject to income tax. D) None of the distribution is taxable or subject to a penalty.
C) $45,000 is subject to the 10 percent early withdrawal penalty and $20,000 is subject to income tax. The distribution is not a qualified distribution, which means that option (D) is not correct. The distribution is not qualified, as it does not meet one of the distribution reasons, which is the attainment of age 59½. The $20,000 of earnings is subject to tax and penalty. The $25,000 of conversions is not subject to tax as it was already subject to income tax. However, because the conversion took place within the last five years, it is subject to the 10 percent penalty.
Contributing $1,500 to his retirement fund at the end of each year beginning at age 18 through age 50, with an average annual return of 12%, how much does Juan have in his retirement account at this time to use toward a possible early retirement? Question 6 options: A) $346,766.42 B) $399,987.65 C) $457,271.58 D) $541,890.55
C) $457,271.58 N = 50 - 18 = 32 i = .12 PV = 0 Pmt = 1,500 FV = <457,271.5789>
This year, Joe, aged 56, received a distribution of $150,000 from his qualified plan. He had a $250,000 adjusted basis in the plan, and the fair market value of the account was $750,000 as of the date of the distribution. Calculate the taxable amount, the non-taxable amount of the distribution, and any applicable penalty. Question 26 options: A) $150,000 non-taxable, $0 taxable, $0 tax penalty B) $100,000 non-taxable, $50,000 taxable, $5,000 tax penalty C) $50,000 non-taxable, $100,000 taxable, $10,000 tax penalty D) $0 non-taxable, $150,000 taxable, $15,000 tax penalty
C) $50,000 non-taxable, $100,000 taxable, $10,000 tax penalty Because the distribution to Joe does not qualify for the exception to the 10% penalty, the taxable amount of the distribution will be subjected to a 10% penalty. To calculate the amount of the distribution that is a return of adjusted basis, the adjusted basis in the plan must be divided by the plan's fair market value as of the day of the distribution. This ratio should then be multiplied by the gross distribution amount. As such, $50,000 of the $150,000 distribution, or ($250,000 ÷ $750,000) × $150,000, is return of adjusted taxable basis and is therefore non-taxable. Accordingly, $100,000 (or $150,000 − $50,000) will be subject to income tax, and there will be a $10,000 tax penalty ($100,000 × 10%).
Charles, a single 29-year-old CEO of a technology start-up company, earns a $2 million base salary with a $400,000 bonus. He is not a participant in any retirement plans at work. What is the maximum deductible IRA contribution Charles can make during 2023? A) $0 B) $1,000 C) $6,500 D) $7,500
C) $6,500 Because he is not an active participant in an employer-sponsored retirement plan, he can contribute and deduct the maximum amount of $6,500 for the 2023 tax year. He is not eligible for a catch-up contribution due to his age.
Abbigail is covered by a group term life insurance policy equal to 5 times her annual salary of $150,000. Her employer pays the entire premium for the policy, for which the uniform annual premium is $0.15 per $1,000 per month of coverage. How much, if any, of the cost of the group term life insurance is included in Abbigail's gross income, assuming 6 months of coverage is provided in the current year? Question 95 options: A) $0 B) $105 C) $630 D) $1,260
C) $630 Abbigail can exclude the cost of up to $50,000 of group term life insurance coverage while the cost of the remaining $700,000 of coverage must be included in gross income. [($750,000 - 50,000 ÷ $1,000) × 0.15] × 6 months equals $630.
Corey, aged 54 and single, has compensation this year of $85,000. His employer does not sponsor a qualified plan, so Corey would like to contribute to a Roth IRA. What is Corey's maximum contribution for this year to the Roth IRA? Question 62 options: A) $0 B) $6,000 C) $7,000 D) $21,250
C) $7,000 The maximum Roth IRA contribution is $6,500 plus $1,000 (2023) for those individuals aged 50 and over. Corey can make a $7,500 contribution to his Roth IRA. Corey's income does not exceed the phase-out limit for 2023.
Callie owns and operates Callie's Creative Coaching (3C), a sole proprietorship. 3C sponsors a 401(k)/profit-sharing plan. Callie had net income of $230,000 and paid self-employment taxes of $20,000 (assumed) during the year. Assuming Callie is over the age of 50, what is the maximum amount that Callie and her company can contribute on her behalf to the plan for 2023? A) $30,000 B) $66,000 C) $73,500 D) $74,000
C) $73,500 $230,000 Net Income ($10,000) Minus 1/2 SE Tax $220,000 Net SE Income x 0.200.25/1.25 $44,000 $22,500 401(k) contribution $7,500 Catch-up $74,000 Total $73,500 Max ($66,000 + $7,500) Callie can contribute a profit-sharing, 401(k), and catch-up contribution. The total cannot exceed the 415(c) limit of $66,000 (2023) plus the catch-up contribution of $7,500.
A company's defined-benefit pension plan utilizes a funding formula that considers years of service and average compensation to determine the pension benefit payable to the plan participants. If Kim is a participant in this defined-benefit pension plan and she has 30 years of service with the company and always had annual compensation of $75,000, what is the maximum pension benefit that can be payable to Kim at her retirement? Question 18 options: A) $19,000 B) $56,000 C) $75,000 D) $245,000
C) $75,000' The maximum amount payable from a defined benefit pension plan is the lesser of $265,000 (2023) or 100% of the average of the employee's 3 consecutive years of highest compensation. Because the average of Kim's compensation is $75,000, she would be limited to receiving a pension benefit at her retirement of $75,000.
Derek is covered by a group term life insurance policy equal to twice his annual salary of $200,000. His employer pays the entire premium for the policy, for which the uniform annual premium is $0.23 per $1,000 per month of coverage. How much, if any, of the cost of the group term life insurance is included in his gross income on an annual basis? Question 50 options: A) $0 B) $92 C) $966 D) $1,104
C) $966 Carlos can exclude the cost of up to $50,000 of group term life insurance coverage, but the cost of the remaining $350,000 of coverage must be included in gross income. [($400,000 − 50,000) ÷ $1,000] × 0.23 × 12 months equals $966.
Which of the following people would be considered a highly compensated employee for 2023? Sloane, a 2% owner whose salary last year was $165,000. Ferris, a 6% owner whose salary was $23,500 for the last five years. Jeanie, an officer, who earned $115,000 last year and is the sixth highest paid employee of 96 employees. Cameron, a 0.5% owner who earned $136,000 last year and is in the top 20% of paid employees. A) 1 and 4. B) 3 and 4. C) 1, 2, and 4. D) 1, 2, 3, and 4.
C) 1, 2, and 4. Sloane and Cameron are HC due to compensation being greater than $135,000 in 2022, the look-back year ($150,000 for 2023). Ferris is HC because he is a >5% owner. Jeanie is not highly compensated because she did not have compensation greater than $135,000 in 2022, the look-back year ($150,000 for 2023).
Beth is the designated beneficiary of a Roth 401(k) plan that she inherited from her dad, Bob, who was 76 years old when he died in 2022. How should Beth calculate the required minimum distribution she will need to take for 2023? Question 49 options: A) Non-recalculating single life expectancy method B) Non-recalculating uniform life expectancy method C) 10-year rule D) Distributions are not required for a Roth 401(k)
C) 10-year rule Based on the SECURE Act, Beth is a non-eligible designated beneficiary of the Roth 401(k) plan account. Since her father died in 2021, she must use the 10-year rule to determine her required minimum distributions. Statement (A) is incorrect. Statement (B) is incorrect because no such method exists. Statement (D) is incorrect because inherited Roth accounts are subject to RMDs.
Which of the following plans is typically subject to a substantial risk of forfeiture as a result of funds being available to creditors upon liquidation of the employer? 1. Secular trust. 2. Rabbi trust. 3. Unfunded promise to pay A) 1 and 2. B) 1 and 3. C) 2 and 3. D) 1, 2, and 3.
C) 2 and 3. Secular trusts are designed to avoid substantial risk of forfeiture. As a result the funding is taxable, to the extent vested.
Marcus has been employed by GCD Enterprises for 15 years, and currently earns $60,000 per year. Marcus saves $15,000 per year. He plans to pay off his home at retirement and live debt free. He currently spends $12,000 per year on his mortgage. What do you expect Marcus' wage replacement ratio to be, based on the above information? Question 3 options: A) 28.41% B) 33.02% C) 47.35% D) 55.00%
C) 47.35% Calculate the wage replacement ratio: Salary $60,000 100.00% Payroll Taxes ($4,590) (7.65%) Savings ($15,000) (25.00%) Mortgage Paid-off ($12,000) (20.00%) $28,410 47.35%
Marigold, aged 50, earns $90,000 per year from her employer. She saves $15,000 per year for retirement and pays $12,000 per year for her home mortgage. Given this information and considering that Marigold will have eliminated her mortgage debt before retirement, what is her expected wage replacement ratio during retirement? Question 2 options: A) 43.16% B) 50.81% C) 62.35% D) 75.68%
C) 62.35% Salary $90,000.00 - 100.00% Payroll Taxes $6,885.00 - 7.65% Savings $15,000.00 - 16.67% Paid-off Mortgage− $12,000.00 - 13.33% $56,115.0062.35%
In which of the following scenarios would a qualified distribution from a Roth IRA have been made? A) A 60-year-old opens her first Roth IRA and 3 years later takes a distribution. B) A 40-year-old takes $10,000 from a 10-year-old Roth IRA to help pay for a second home. C) A 35-year-old becomes totally disabled and distributes $100,000 from a 6-year-old Roth IRA. D) A 55-year-old with a 30-year-old Roth distributes $5,000 to pay his daughter's preschool tuition.
C) A 35-year-old becomes totally disabled and distributes $100,000 from a 6-year-old Roth IRA. In order to be considered a qualified distribution, a Roth IRA must be open for at least 5 years and one of the following must be true: the account holder is at least 59½ years old, the account holder is dead and the beneficiary receives the distribution, the account holder is disabled, or the account holder is taking out up to $10,000 to help pay for a first-time home purchase. Answer (A) is incorrect because the Roth has not been open for 5 years. Answer (B) is incorrect because the distribution is for a second home. Answer (D) is incorrect because paying for preschool education is not considered a qualified distribution, though it does not incur a 10% penalty as an exception to 72t.
In which of the following plans may an employer make tax-deductible contributions to a trust in the form of both principal and interest for a loan? Question 41 options: A) A stock bonus plan B) An ESOP C) A leveraged ESOP D) An S corporation ESOP
C) A leveraged ESOP Only a leveraged ESOP will have a loan and thus have principal and interest payments.
Allison Quinn, aged 43, is an employee of BizzCo Corporation and earns an annual salary of $95,000. Allison has recently been informed that the company is implementing a group term-life-insurance plan and the company will pay the premium for an amount of coverage equal to her annual salary. What are the tax implications of this benefit for Allison, assuming 12 months of coverage are provided in the current year? Question 94 options: A) Allison will not be taxed on the benefit as long as the plan is nondiscriminatory. B) Allison will pay tax on an additional $4.50 of income. C) Allison will pay tax on an additional $54.00 of income. D) Allison will pay tax on an additional $114.00 of income.
C) Allison will pay tax on an additional $54.00 of income. Employees are taxed on group term-life-insurance premiums paid for by the employer on death benefits over $50,000. The amount of additional income is calculated using the Table 1 cost of insurance, which shows the monthly cost per $1,000 of death benefit. Since Allison is 43 years old, the monthly cost from Table 1 is 10 cents per $1,000 of death benefit over $50,000. The employer is paying for an extra $45,000 of coverage: 45 × 0.10 × 12 = $54.
Which of the following statements about required notifications is correct? 1. Employers are required to provide, free of charge, a summary of the details of the qualified retirement plan, called a summary plan description, to employees, participants, and beneficiaries under pay status (receiving benefits). The summary must be furnished within 90 days after the person becomes a participant. 2. Employers are required to provide the plan participants notices of any plan amendments or changes. This notice can be provided either through a revised summary plan description or in a separate document, called a summary of material modifications. This document must be given to participants free of charge within 210 days after the end of the plan year in which a change is adopted and applies when there are substantive changes in the plan. A) 1 only. B) 2 only. C) Both 1 and 2. D) Neither 1 nor 2.
C) Both 1 and 2.
Which of the following are requirements for a qualified stock-bonus plan? I. Participants must have pass-through voting rights for stock held by the plan.II. Participants must have the right to demand employer securities at a distribution, even if the plan sponsor is a closely held corporation. Question 36 options: A) I only B) II only C) Both I and II D) Neither I nor II
C) Both I and II
Which of the following statements concerning profit-sharing plans is (are) correct? I. The employer can choose from a number of different allocation formulas.II. Profit-sharing plans are popular because contributions may be made on a discretionary basis. Question 34 options: A) I only B) II only C) Both I and II D) Neither I nor II
C) Both I and II
Factors affecting retirement planning decisions are many and varied. Which of the following statements is correct with regards to making decisions concerning retirement? I. Retirement planning decisions might include factors such as work life expectancy, remaining work life expectancy and retirement life expectancy.II. Retirement planning decisions might include factors such as savings, annual income needs, wage replacement ratios, and inflation. Question 1 options: A) I only B) II only C) Both I and II D) Neither I nor II
C) Both I and II All of the factors listed above can affect retirement decisions.
Which of following statements is (are) correct? I. An individually designed qualified plan will generally cost more for a company to establish than a prototype plan. II. A plan sponsor has the right to terminate a qualified retirement plan when the plan sponsor no longer has the funds available to continue funding the plan at a reasonable level. Question 56 options: A) I only B) II only C) Both I and II D) Neither I nor II
C) Both I and II An individually designed qualified plan will be drafted by attorneys to meet the specific needs and desires of the plan sponsor. A prototype plan is a standard or form plan that has already received a determination letter from the IRS. The plan sponsor simply chooses the appropriate elections for its plan. Because the individually designed qualified plan utilizes attorneys and the plan sponsor often requests a determination letter from the IRS, the costs are higher than those associated with a prototype plan. A plan sponsor can terminate a plan when the plan sponsor can no longer make contributions to the plan. In this case, all participants will become 100% vested in their accrued benefit.
Which of the following statements is (are) true? I. Participants of a stock-bonus plan sponsored by a C corporation must be given pass-through voting rights for the employer stock held by the plan.II. One disadvantage of an ESOP is that it may place an unnecessary cash-flow burden on the plan sponsor. Question 40 options: A) I only B) II only C) Both I and II D) Neither I nor II
C) Both I and II Both statements are correct. Participants of a stock-bonus plan sponsored by a C corporation must be given pass-through voting rights for the employer stock held by the plan. An ESOP can create an unnecessary cash-flow burden for the plan sponsor in any of the following cases: (1) a participant terminates employment and requests a distribution of cash from the plan, (2) a participant attains the age of 55 and requests diversification within the plan, (3) the plan sponsor must pay for a periodic appraisal of the employer stock, and (4) a participant exercises his put/repurchase option.
Employee Stock Ownership Plans (ESOPs) are a special kind of Stock Bonus plan. Which of the following statements reflect a feature or benefit of an ESOP? I. ESOPs are design to primarily invest in employer stock.II. ESOPs are funded solely with employer contributions, as no CODAs are allowed. Question 20 options: A) I only B) II only C) Both I and II D) Neither I nor II
C) Both I and II ESOPs are designed primarily to invest in employer stock and they are funded exclusively by employers, as no CODA are permitted.
Group benefits generally offer the employee the opportunity to obtain various insurance and related benefits at lower rates than are available to individuals outside of the group. Which of the following are considered advantages of group benefits? I. Group benefits are usually not included in taxable income of the employeeII. Employers can deduct the costs of provided group benefits, assuming certain conditions are met Question 47 options: A) I only B) II only C) Both I and II D) Neither I nor II
C) Both I and II In addition to these two features, group benefits usually have lower rates and provide better coverage than would be available to employees pursuing them individually.
When considering a Simple 401(k) plan, which of the following statements is true? I. This type of plan can be better structured to maximize benefits for the owner.II. This type of plan can be structured to maximize benefits for the owner's spouse if employed by the sponsoring organization. Question 36 options: A) I only B) II only C) Both I and II D) Neither I nor II
C) Both I and II Single participant plans, like Simple 401(k)s, can be structured to benefit both the owner and owner's spouse.
The bucket approach retirement income withdrawal strategy has the following advantage(s): I. The overall portfolio from the combined tranches is similar to a moderately allocated portfolio.II. This strategy helps the client focus on and differentiate current needs from future needs. Question 9 options: A) I only B) II only C) Both l and ll D) Neither l nor ll
C) Both l and ll The Money-for-Life withdrawal strategy recognizes that retirees have both short-term and long-term needs. So, the first tranche might invest 25% of the portfolio in money market funds for the first 5 years, while the tranche for years 26-30 might have 12% of the portfolio invested in small-cap stocks in order to provide growth. However, combining the various tranches, the overall portfolio would be similar to a moderately allocated portfolio, much like the 4% per year approach.
Which of the following is subject to IRC Section 409A? A) ESOPs. B) ESPPs. C) ISO plans. D) 457(f) plans.
C) Constructive receipt. Substantial risk of forfeiture deals with a risk of a payment not being made and its impact on taxability. Tax triads do not have anything to do with deferred compensation. The economic benefit doctrine provides that an employee will be taxed on funds or property set aside for the employee if the funds or property are unrestricted and nonforfeitable.
Dr. Morgan, age 62, is a participant in a stock bonus plan. The value of the employer stock contributions to the plan over the course of her participation totaled $100,000. On December 1st of last year, Dr. Morgan decides to execute an NUA strategy and takes a full distribution of the employer stock from the plan at a value of $350,000. 6 months later in the current year, Dr. Morgan sells all of the stock for $300,000. Which of the following statements is true? Question 38 options: A) Dr. Morgan has a long-term capital gain of $250,000 for last year. B) Dr. Morgan has ordinary income of $350,000 for last year. C) Dr. Morgan has a long-term capital gain of $200,000 in the current year. D) Dr. Morgan has a short-term capital gain of $200,000 in the current year.
C) Dr. Morgan has a long-term capital gain of $200,000 in the current year. Because Dr. Morgan is taking a lump-sum distribution from a qualified plan of employer stock, she will not have to recognize the net unrealized appreciation until she disposes of the employer stock. However, at the time of the distribution, the value of the stock, as of the date of contribution to the plan, will be taxable as ordinary income. Any gain on the subsequent sale of the stock will be taxable as long-term capital gain. In this case, Dr. Morgan will recognize $100,000 of ordinary income at the date of the distribution and long-term capital gain of $200,000 (that is, $300,000 − $100,000) at the date of sale.
On January 15, Year 1, Oscar, a Senior Vice President for Meyer Corporation, is granted 20,000 ISOs at an exercise price of $10. On February 6, Year 2, he exercises all his options when the price of Meyer stock is $27. When can Oscar sell the ISO shares and avoid a disqualifying disposition? A) January 16, Year 3. B) January 16, Year 4. C) February 7, Year 3. D) February 7, Year 4.
C) February 7, Year 3. For a qualifying disposition, he must wait two years from the date of grant and one year from the date of exercise.
Which of the following statements regarding 457(b) plans is (are) true? I. An individual who defers $22,500 to his 403(b) plan during 2023 can also defer $22,500 to a 457(b) plan during 2023 (salary and plan permitting).II. A 457(b) plan allows an executive of a tax-exempt entity to defer compensation into an ERISA protected trust.III. In the final three years before normal retirement age, a participant of a government sponsored 457(b) plan may be able to defer $45,000 (2023) for the plan year. Question 71 options: A) I only B) II only C) I and III D) II and III
C) I and III Statement II is false. The funds deferred to a 457(b) plan established for a tax-exempt entity do not have ERISA protection. Statement I is true because the deferral limits for 403(b) plans and 457(b) plans are separate. Statement III is true as certain 457(b) plans allow the participants to defer twice the annual deferral limit in the last three years before the plan's normal retirement age.
Which of the following investments cannot be used to fund a 403(b) plan? Question 37 options: A) Fixed annuity contracts B) Variable annuity contracts C) Individual stocks and bonds D) Mutual funds
C) Individual stocks and bonds Individual stocks and bonds, along with life insurance, are not investments permitted to fund 403(b) accounts.
All of the following are reasons why an employer might favor a nonqualified plan over a qualified retirement plan EXCEPT: Question 78 options: A) There is more design flexibility with a nonqualified plan. B) A nonqualified plan typically has lower administrative costs. C) Nonqualified plans typically allow the employer an immediate income tax deduction. D) Employers can generally exclude rank-and-file employees from a nonqualified plan.
C) Nonqualified plans typically allow the employer an immediate income tax deduction. Nonqualified retirement plans do not allow the employer to take an income tax deduction until the employee recognizes the income. All of the other statements are correct.
Oliver is a 45-year-old executive who earns $375,000 from his job at Acme Arrows (AA) and contributes the maximum amount to the 401(k) plan. He wants to make a contribution to a Roth IRA for the current year. Assume that Oliver has a traditional IRA with a balance of $10,000 that was funded entirely with pre-tax contributions. Which of the following is correct for 2023? Question 64 options: A) Oliver cannot contribute to a Roth IRA because he is an active participant in his company's retirement plan. B) Oliver cannot execute the backdoor Roth strategy because he has a balance in a traditional IRA. C) Oliver could execute the backdoor Roth strategy but would have to recognize approximately $2,560 for income tax purposes if he contributes $6,500 to a traditional IRA. D) If Oliver executes a backdoor Roth strategy, he has the option of recharacterizing the conversion if the value of the assets decreased prior to the fil
C) Oliver could execute the backdoor Roth strategy but would have to recognize approximately $2,560 for income tax purposes if he contributes $6,500 to a traditional IRA. Oliver could execute a backdoor Roth but would have to recognize approximately $3,750 into income: 1 - ($6,500 / $16,500) × $6,500 = $2,560. Recharacterization of conversions is no longer permitted after 2017 as a result of the TCJA 2017.
Which of the following statements is true? Question 32 options: A) Profit-sharing plans may not offer in-service withdrawals. B) Pension and profit-sharing plans are both subject to mandatory funding requirements. C) Profit-sharing plans allow annual employer contributions up to 25 percent of the employer's covered compensation. D) The legal promise of a profit-sharing plan is to pay a pension at retirement.
C) Profit-sharing plans allow annual employer contributions up to 25 percent of the employer's covered compensation. Statement (C) is the only true statement. Profit sharing plans allow annual contributions of up to 25 percent of covered compensation. Statement (A) is false because profit-sharing plans can allow in-service withdrawals. Statement (B) is false because while pension plans are subject to mandatory funding standards, profit-sharing plans are not. Statement (D) is false because the legal promise of a profit-sharing plan is the deferral of compensation, and the legal promise of a pension plan is to a pay a pension at retirement.
The following statements are true with regards to an IRC 127 educational assistance program EXCEPT Question 92 options: A) Qualifying expenses include undergraduate and graduate tuition. B) Qualifying expenses include qualifying student loan payments. C) Qualifying expenses include K-12 tuition. D) Up to $5,250 of expenses may be excluded from the employee's gross income under this plan.
C) Qualifying expenses include K-12 tuition. Qualifying expenses include undergraduate and graduate expenses and qualifying student loan payments. Payments made for K-12 tuition are not considered qualifying expenses under IRC 127.
All of the following people would be considered a highly compensated employee for 2023 EXCEPT Question 10 options: A) Kim, a 1% owner whose salary last year was $150,000 B) Rita, a 6% owner whose salary was $42,000 last year C) Robin, an officer, who earned $125,000 last year and is the 29th highest paid employee of 96 employees D) Helen, who earned $160,000 last year and is in the top 20% of paid employees
C) Robin, an officer, who earned $125,000 last year and is the 29th highest paid employee of 96 employees Kim and Helen are HC due to compensation being greater than $135,000. Rita is HC because she is a > 5% owner. Robin is not highly compensated because she does not have compensation greater than $135,000 (2023).
Fred, aged 30, has a small but promising business with 5 current employees. He wishes to adopt an easily managed workplace plan that would allow himself and his employees to defer modest amounts of their salary into the plan. Fred is not interested in maximizing his own deferrals because he is plowing most of the earnings back into the business. He is concerned, however, about maintaining plan choice flexibility in case his business suddenly expands in size. Which of the following plans would Fred prefer to adopt? Question 74 options: A) SEP IRA B) Profit sharing with 401(k) C) SIMPLE IRA D) Target-benefit pension plan
C) SIMPLE IRA A SIMPLE IRA would be ideal for Fred's business. Option (A) is incorrect because SEP IRAs do not allow employee-salary deferrals. Option (B) is incorrect because the additional complexity to maintain a qualified plan and higher deferral limits do not match Fred's requirements. Option (D) is incorrect because target benefit pensions do not depend on employee salary deferrals.
At the beginning of this year, Sophia contributed $3,000 to her HSA. She invested that contribution into stocks and, by the end of the year, her account balance grew to $4,000. Which of the following statements correctly describes the tax treatment of this contribution and subsequent gain? A) The full $4,000 will be included in her gross income but she will be able to make a $3,600 above-the-line deduction for the current tax year. B) $3,600 will be included in her gross income but will be negated by a $3,600 above-the-line deduction; the remaining $500 will be subject to capital gains tax treatment for the current tax year. C) She will be able to make a $3,000 above-the-line deduction and the $1,000 gain will not be included in her gross income or subject to capital gains tax treatment for the current tax year. D) She will be able to make a $3,000 above-the-line deduction and the $1,000 gain will be subject to capi
C) She will be able to make a $3,000 above-the-line deduction and the $1,000 gain will not be included in her gross income or subject to capital gains tax treatment for the current tax year.
Qualified plans must satisfy many tests to maintain qualified status. Which of the following is correct regarding coverage tests? A) Defined-benefit (DB) plans must pass any two of the four coverage tests. B) Coverage testing never includes leased employees as part of the calculation. C) Stock bonus plans must satisfy one of three coverage tests only. D) Employees who do not meet the eligibility requirements are still included in the determination of at least one of the coverage tests.
C) Stock bonus plans must satisfy one of three coverage tests only. Options (A), (B), and (D) are incorrect. Qualified plans must pass one of the three coverage tests, and DB plans must pass one of the three tests and the 50/40 test. The 50/40 test is based on total employees, not necessarily non-highly compensated employees. All coverage tests exclude non-eligible employees from the calculation. Leased employees may have to be included in the coverage calculation if they meet certain requirements.
All the following statements concerning Voluntary Employee Beneficiary Associations (VEBAs) are correct EXCEPT: Question 93 options: A) The employer may deduct contributions for federal income tax purposes. B) The employer avoids taxation on the investment income earned on contributions. C) The employee may be provided retirement and miscellaneous fringe benefits. D) Noncurrent employees may become members of a VEBA.
C) The employee may be provided retirement and miscellaneous fringe benefits. Employees may not be provided retirement or miscellaneous fringe benefits. Statements (A), (B), and (D) are correct statements. Note that the number of noncurrent employees that may become members of a VEBA cannot exceed 10% of the total membership.
Which of the following applies to key-person life insurance? Question 97 options: A) The premiums are tax-deductible. B) The employee is the beneficiary. C) The employer pays the cost. D) The employee is the policy owner.
C) The employer pays the cost The premiums are not tax-deductible. The employer is the owner, premium payer, and beneficiary. Therefore, statements (A), (B), and (D) are incorrect.
All of the following are reasons for a qualified plan to be amended or terminated EXCEPT: Question 29 options: A) In response to changes in applicable laws or regulations B) The employer is unable to financially support plan contributions C) The plan is meeting the needs of the company and/or employees D) The benefits provided to employees are not sufficient to justify plan costs
C) The plan is meeting the needs of the company and/or employees If a plan is meeting the needs of the employer and employees, no changes would be needed. If the plan was not meeting the needs of employer and /or employees, it would be a reason to amend or terminate a plan. (A), (B), and (D) are all reasons to amend or terminate a qualified plan.
All of the following statements about ESOPs are correct EXCEPT: Question 42 options: A) An ESOP is controlled through a trust. B) ESOPs provide corporate owners with a way to transfer ownership interests to their employees. C) The trust of an ESOP is prohibited from borrowing money from a bank to purchase the employer stock. D) Some conflicts of interests among employees, the corporation, and management of the corporation do not require the ESOP trustee to resign.
C) The trust of an ESOP is prohibited from borrowing money from a bank to purchase the employer stock. A key characteristic of the ESOP is that the trust may borrow money to purchase the employer stock.
Which of the following plans needs an actuary on an ongoing basis? Question 10 options: A) Tandem plan. B) Cash balance plan. C) 412(e) plan. D) Target benefit plan.
Cash balance plan. The tandem plan consists of a money purchase pension plan and a profit sharing plan. The cash balance plan needs an actuary. The 412(e) plan is fully funded with insurance and does not need an actuary. The target benefit plan does not need an actuary after it is set up.
All of the following statements are correct regarding simplified employee pension plan (SEP) contributions made by an employer, EXCEPT: Contributions are subject to taxation under the Federal Insurance Contributions Act (FICA) and the Federal Unemployment Tax Act (FUTA). Contributions are currently excludable from the employee-participant's gross income. Contributions must be made to all employees who were eligible that year - even if they are no longer employed at the time of the employer contribution to the SEP IRA. Contributions cannot be more than 25 percent of compensation for employees.
Contributions are subject to taxation under the Federal Insurance Contributions Act (FICA) and the Federal Unemployment Tax Act (FUTA). Employer contributions to a SEP are not subject to FICA and FUTA.
Bob broke his leg skiing earlier this year. Bob collected $10,000 from his short-term disability plan paid for solely by his employer. How much does Bob have to include in his gross income? Question 98 options: A) $0 B) $5,000 C) $8,500 D) $10,000
D) $10,000 Bob must include 100% of the disability premiums since his employer paid the entire premium.
Jennifer, aged 54, earns $125,000 annually from ABC Incorporated. ABC sponsors a SIMPLE and matches all employee deferrals 100% up to a 3% contribution. Assuming Jennifer defers the maximum to her SIMPLE, what is the total contribution to the account in 2023 including both employee and employer contributions? Question 69 options: A) $17,000 B) $17,250 C) $22,500 D) $22,750
D) $22,750 Jennifer can defer up to $19,000 ($15,500 + $3,500) for 2023 because she is over 50. ABC's match for Jennifer is 3% of her compensation, or $3,750 (3% × $125,000). The maximum contribution to Jennifer's SIMPLE is $22,750 ($15,500 + $3,500 + $3,750).
Paper Mill, Inc. provides a fringe benefit valued at $1,000 to its rank-and-file employees and a similar fringe benefit valued at $3,000 to its officers and highly compensated employees. The IRS has deemed this benefit to be discriminatory. Consequently, how much must each officer and highly compensated employee include in his gross income? Question 91 options: A) $0 B) $1,000 C) $2,000 D) $3,000
D) $3,000 The plan is discriminatory to non-highly compensated employees; therefore, the entire value of the fringe benefit given to officers and highly compensated employees is includible in their gross income, not just the excess of what is available to the other employees.
Axe company sponsors a 401(k) profit sharing plan with no employer match, but the company did make noncontributory employer contributions because the plan was top-heavy. Greta quit today after six years working for Axe and has come to you to determine how much of her retirement balance she can take with her. The plan uses the least generous graduated vesting schedule available. What is Greta's vested account balance if she has been a participant for 57 months? EMPLOYER $9,000 EMPLOYEE CONTRIBUTIONS $12,000 EARNINGS $4,000 $5,000 A) $17,000 B) $24,800 C) $27,400 D) $30,000
D) $30,000 Greta is entitled to 100% of her contributions and the earnings on those contributions. The employer contributions, which were not matching contributions, will follow the least generous graduated vesting schedule for a top-heavy plan. The least generous graduated vesting schedule is a 2-to-6-year graduated vesting schedule for a 401(k) plan. At six years of service, Greta is 100% vested in the employer contributions and the earnings on the employer contributions. Thus, Greta's vested account balance is $30,000.
Caroline, age 55, is a public school administrator whose system offers both a 457(b) and a 403(b) plan to employees. What is the maximum she can contribute to these plans in 2023? Question 76 options: A) A total of $22,500 between the 457(b) and 403(b) B) A total of $30,000 between the 457(b) and 403(b) C) $22,500 to the 403(b) and $22,500 to the 457(b) D) $30,000 to the 403(b) and $30,000 to the 457(b)
D) $30,000 to the 403(b) and $30,000 to the 457(b) $60,000 or $30,000 each to both the 457(b) and 403(b) plans. The salary deferral limits are not shared between 457(b) and 403(b) plans unlike they would be for an employee eligible to participate in both a 401(k) and 403(b) plan.
Perry operates In-N-Out Pharmacy, a sole proprietorship. In-N-Out sponsors a profit-sharing plan. Perry had net income of $205,000 and ½ of his self employment taxes was $15,000 for the year. If Perry makes a 25% of salary contribution on behalf of all of his employees to the profit-sharing plan, how much is the contribution to the profit-sharing plan on behalf of Perry? Question 51 options: A) $22,820 B) $24,776 C) $28,500 D) $38,000
D) $38,000 $205,000 Net Income ($15,000) Less 1/2 SE Tax $190,000 Net SE Income × 0.20 - 0.25/1.25 $38,000 Contribution
Max earned $70,000 last year and made $22,500 in CODA deferrals. He received a $5,000 match from his employer. If his employer also wants to make a year-end profit-sharing contribution to Max's account, it must not exceed what amount for 2023? Question 16 options: A) $6,500 B) $26,000 C) $30,000 D) $38,500
D) $38,500 Max contribution limit: $66,000 - $22,500 - $5,000 = $38,500
Lisandro, aged 25, is looking to retire in 35 years when he turns 60. He wants to have enough money to replace 80% of his current income less what he expects to receive from Social Security at the beginning of each year. He expects to receive $30,000 per year from Social Security in today's dollars at age 60 (assuming this is permissible when he retires). Lisandro wants to assume a 7% annual investment rate of return while also assuming that inflation will be 4% per year. He would like to plan for a retirement with a savings amount that lasts until He is 100 years old. If he currently earns $100,000 per year, approximately how much does he need at retirement to fulfill his retirement goals? Question 3 options: A) $4,276,989.21 B) $4,478,979.68 C) $4,646,935.63 D) $4,780,981.14
D) $4,780,981.14 The answer is calculated by completing the following steps. Note that answers may vary slightly due to rounding. Answer (C) is the answer students will arrive at if they fail to select Begin Mode on their calculator. Step 1: Use the following figures to determine the future value of the money that Lisandro will need to replace: PV = $50,000 = (0.80 × $100,000) − $30,000 N = 35 i = 4% PMT = $0 The future value is $197,304.45. Step 2: Determine the present value of that amount: BEGIN MODE FV = 0 N = 40 i = 2.8846% = [(1.07 ÷ 1.04) − 1] × 100 PMTAD = $197,304.45 The present value of that amount is $4,780,981.14.
Marisol was granted 100 NQSOs five years ago. At the time of the option grant, the value of the underlying stock was $100 and the exercise price was equal to $100. If Marisol exercises the options on August 22 of this year when the stock is valued at $145, what are the tax consequences (per share) to Marisol from exercising the options? Question 77 options: A) $45 of W-2 income, $100 of short-term capital gain B) $100 of W-2 income, $45 of short-term capital gain C) $145 of W-2 income D) $45 of W-2 income
D) $45 of W-2 income At the exercise date of an NQSO, the individual will have to buy the stock at the exercise price and will have W-2 income for the stock value's appreciation amount in excess of the exercise price. In this case, Marisol will have $45 ($145 - $100) of W-2 income. There is no other gain or loss at exercise.
Dr. Durr has worked as a professor at Top-Notch University for the last 30 years. She has never deferred any money into her 457(b) plan. Dr. Durr has maxed out her 403(b) plan. She will attain normal retirement age under the 457(b) plan in 2023. How much can Dr. Durr contribute to her 457(b) plan for2023? A) $0. B) $22,500. C) $30,000. D) $45,000.
D) $45,000. Dr. Durr is allowed to defer an additional $22,500 in 2023 because she has unused prior deferrals.
Jamie has a traditional IRA with $15,000 of nondeductible contributions and $5,000 of earnings. If she converts her traditional IRA to a Roth IRA, what amount of income will be included in her gross income? Question 34 options: A) $20,000 B) $15,000 C) $10,000 D) $5,000
D) $5,000 A conversion of a traditional IRA to a Roth IRA requires that the taxpayer include the deferred income in the IRA as taxable income in the year of the conversion.
Billy's company sponsors a 401(k) profit-sharing plan with no employer match, but the company did make noncontributory employer contributions because the plan was top-heavy. Billy quit today after six years with the company and has come to you to determine how much of his retirement balance he can take with him. The plan uses the least generous graduated-vesting schedule available. What is Billy's vested account balance? Employer - Employee Contributions $2,000 - $2,000 Earnings $600 - $600 A) $2,600 B) $4,160 C) $4,680 D) $5,200
D) $5,200 Billy is entitled to 100% of his contributions and the earnings on those contributions. The employer contributions, which were not matching contributions, will follow the least generous graduated-vesting schedule for a top-heavy plan. The least generous graduated vesting schedule is a 2- to 6-year graduated-vesting schedule for a 401(k) plan. At six years, Billy would be 100% vested in the employer contributions and the earnings on the employer contributions. Thus Billy's vested account balance is $5,200.
Jan, a 72-year-old widow, is a participant of the Ace Web Enterprise (AWE) 401(k) plan. For the current year, she deferred $6,000, or 10 percent, of her salary to the plan. She did not receive a profit-sharing contribution, a forfeiture, an employer match, or any other employer contribution. Jan would like to make a contribution to her Roth IRA. If Jan's AGI is $75,000 (all comprised of W-2 earnings, Social Security and portfolio income), what is the maximum contribution she can make for 2022? Question 60 options: A) $0 B) $3,000 C) $6,500 D) $7,500
D) $7,500 Her income does not exceed the phase-out for contributing to a Roth IRA. Therefore, she can contribute the maximum amount, including the catch-up.
Marie is planning for her retirement. She is currently 30 years old and plans to retire at age 65. She expects to live until age 105. She currently earns $75,000 per year and anticipates she'll need 75% of her income during retirement. She anticipates Social Security will provide her with $20,000 per year at age 65, leaving her with required savings to provide $36,250, that is, $75,000 × 0.75 − $20,000, annually during retirement. She believes she can earn 9% on her investments and that inflation will be 3% per year. How much must Marie save at the end of each year if she wants to make her last savings payment at age 65 and she does not have any legacy or bequest goals (rounding is assumed)? Question 5 options: A) $5,250 B) $6,237 C) $7,235 D) $7,698
D) $7,698 Step 1: Use the following figures to determine the future value of the money that Marie will need to replace: PV = $36,250 = (0.75 × $75,000) − $20,000 Step 2: Inflate the need to the beginning of retirement. PV: <$36,250> I/YR: 3 N: 35 PMT: 0 FV: $102,002.51 Step 3: Determine the funding need at retirement age. BEGIN MODE PMT: $102,002.51 I/YR: [(1.09 ÷ 1.03) − 1] × 100 = 5.8252 N: 40 FV: 0 PV: <$1,660,605.21 Step 4: Determine the required annual savings amount .END MODE FV: $1,660,605.21 N: 35 I/YR: 9 PV: 0 PMT: <$7,698.30
James is employed by a large corporation with 400 employees. The corporation provides its employees with a no-cost gym membership at the local public YMCA. The cost of the membership is $60/month, which is completely paid for by James' employer for all employees. How much, if any, must James include in his yearly gross income related to this fringe benefit? Question 85 options: A) $0 B) $60 C) $600 D) $720
D) $720 James must include the full cost paid by his employer in his adjusted gross income. The exclusion for payment of health club facility dues is only provided when the facilities are on the employer's business premises and are solely for the use of the employees and their families. In this example, James' employer provides a membership at a public YMCA, so the fringe benefit is taxable.
Elsie receives a lump-sum distribution of employer securities (1,000 shares) from her stock bonus plan in Year 1 worth $140,000. The NUA for the stock equals $110,000 at the time of the distribution. Elsie sells half of the shares four years after she received it as a distribution from the qualified plan. She receives proceeds of $100,000 from the sale. How much is Elsie's capital gain? A) $55,000 B) $65,000 C) $75,000 D) $85,000
D) $85,000 The tax basis for the 1,000 share is $30,000, the difference between the NUA and the proceeds. If she sells half the shares for $100,000, then the basis for that half equals $15,000. The LTCG equals $85,000.
Jason turned 72 in November of this year, which serves as the required beginning date (RBD). He was a participant in his employer's profit-sharing plan. His profit-sharing plan had an account balance of $250,000 on December 31 of last year. According to the Uniform Lifetime Table, the factors for ages 70, 71, and 72 are 27.4, 26.5, and 25.6 respectively. What is Jason's required minimum distribution for this year? Question 47 options: A) $7,300 B) $8,547 C) $8,812 D) $9,766
D) $9,766 Jason must take a required minimum distribution for this year because he turned 72. The distribution is calculated by taking the account balance at the end of last year divided by the factor for Jason's age at the end of this year. $250,000 / 25.6 = $9,766.
All of the following are permitted vesting schedules for a non-top-heavy profit-sharing plan EXCEPT: Question 27 options: A) 2-to-6 year, graduated B) 3-year, cliff C) 1-to-4 year, graduated D) 3-to-7 year, cliff
D) 3-to-7 year, cliff As a result of the Pension Protection Act of 2006, a profit-sharing plan must vest at least as rapidly as a 3-year cliff, or 2-to-6-year graduated schedule without regard to the plan's top-heavy status. The profit-sharing plan can follow any vesting schedule that provides a more generous vesting schedule.
Which of the following is subject to IRC Section 409A? A) ESOPs. B) ESPPs. C) ISO plans. D) 457(f) plans.
D) 457(f) plans. Qualified plans, ESPPs, ISOs are all excluded from coverage, in most cases. 457(f) plans are subject to 409A.
Participation in which of the following plans will not be considered active participation for determining the deductibility of traditional IRA contributions? A) A tax-sheltered annuity B) A simplified employee pension C) A SIMPLE D) A 457(b) plan
D) A 457(b) plan The 457(b) plan is not considered a retirement plan under the IRA deductibility rules.
Which of the following statements is true? Question 23 options: A) A cash-balance pension plan usually benefits older employees the most. B) A defined-benefit plan promises a contribution to a hypothetical account each year for a plan participant. C) Cash-balance pension-plan participants under the age of 59 ½ years may take a withdrawal from the plan during employment with the plan sponsor. D) A cash-balance pension plan does not have individual separate accounts for each participant.
D) A cash-balance pension plan does not have individual separate accounts for each participant. Statement (D) is true. The cash-balance pension plan consists of a commingled account that must be equal to the actuarial equivalent of the present value of the expected future benefits that will be paid from the cash-balance pension plan. Statement (A) is false because cash-balance pension plans usually benefit younger participants because of the guaranteed contribution rate and the guaranteed earnings rate. Statement (B) is false because a defined-benefit plan promises a defined benefit at the participant's retirement. If the funding requirements are met with plan earnings, a contribution is not required. Statement (C) is false because the participant of a cash-balance pension plan under the age of 59 ½ years cannot take in-service withdrawals.
When calculating the plan contribution in a given year for a Defined Benefit Plan, an actuary might decrease the proposed funding amount based on which of the following factors: Question 26 options: A) Lower than expected plan forfeitures B) Investment returns that are lower than anticipated C) Lower employee turnover D) A decrease in expected inflation
D) A decrease in expected inflation All other answers would have the opposite effect on plan funding.
What is the first year in which a single taxpayer, at age 48 in Year 2, could receive a qualified distribution from a Roth IRA if he opened a Roth IRA for the first time on April 1 of Year 2 for the tax year prior, and assuming neither the death nor disability exceptions apply? Question 63 options: A) Year 5 B) Year 6 C) Year 7 D) After he turns 59½
D) After he turns 59½ A qualified distribution can only occur after a five-year period has occurred and is made on or after the date on which the owner attains age 59½, the owner's death or disability, or for a first-time home purchase. The five-year period starts at the beginning of the taxable year of the initial contribution to a Roth IRA. The five-year period ends on the last day of the individual's fifth consecutive taxable year, beginning with the taxable year described in the preceding sentence.
Which of the following factors may affect a retirement plan? I. Career earningsII. Retirement life expectancyIII. MortalityIV. Savings rate Question 4 options: A) I and II B) II and III C) I, II, and IV D) All of the above
D) All of the above All of the options may affect a retirement plan either positively or negatively. Reduced work life expectancy may provide an insufficient savings period while an increased retirement life expectancy increases capital needs for retirement. A low savings rate may create an inability to meet capital requirements. Increased inflation rates will reduce purchasing power. The career earnings will affect the retirement need. As mortality increases, less is needed at retirement.
Which of the following statements reflects a mandatory requirement of pension plans: I. Only defined benefit pension plans need to offer a qualified joint and survivor annuity (QJSA) to married plan participants.II. Only defined benefit pension plans need to offer a qualified pre-retirement survivor annuity (QPSA) to a plan participant.III. All pension plans must comply with anti-cutback rules that forbid a plan sponsor from amending a plan such that it would serve to reduce a benefit that a participant has already accrued. Question 10 options: A) I only B) II only C) III only D) All of the statements are correct.
D) All of the statements are correct. Statements I, II, and III apply to all pension plans.
Nondiscrimination rules apply to fringe benefits being offered to employees. However, in certain circumstances nondiscrimination rules may not apply. Nondiscrimination rules do not apply to fringe benefits in all of the following examples EXCEPT: Question 45 options: A) The employee may not have reached a certain age. B) A benefit was not offered as part of a collective bargaining agreement. C) An employee is simply not eligible for dependent care being offered. D) An employee does not qualify because she is a full-time hourly worker.
D) An employee does not qualify because she is a full-time hourly worker. Answer choices (A), (B), and (C) are all examples where nondiscrimination rules do not apply to fringe benefits. Answer choice (D) would be discriminatory in that the employee is an hourly worker, not a salaried employee and that would be deemed to be discriminatory.
All of the following are characteristics of non-qualified deferred compensation plans EXCEPT: Question 39 options: A) A substantial risk of forfeiture is present. B) There is no constructive receipt. C) There is an unfunded promise to pay. D) Assets in a "rabbi" trust are protected from sponsoring entities' creditors.
D) Assets in a "rabbi" trust are protected from sponsoring entities' creditors. Answer choices (A), (B), and (C) are all characteristics of non-qualified deferred compensation plans. Answer choice (D) is not a characteristic as rabbi trusts only protect against a change of heart of the sponsor not from insolvency. A secular trust would protect against insolvency claims, but could cause immediate taxation to the employee depending on the type of secular trust that was employed.
Lauren received nonqualified stock options (NQSOs) with an exercise price equal to the fair market value (FMV) of $25 at the date of the grant. She exercised the options 3 years after the grant date, when the FMV of the stock was $30. Lauren then sells the stock for $35 3 years after exercising. Which of the following statements is true? Question 41 options: A) At the date of the grant, she will have ordinary income of $25. B) At the date of exercise, she will have W-2 income of $30. C) At the date of the sale, she will have W-2 income of $35. D) At the date of sale, she will have long-term capital-gain of $5.
D) At the date of sale, she will have long-term capital-gain of $5. Lauren would not have any taxable income at the date of grant provided the exercise price was equal to the fair market value of the stock. Her long-term capital gain is $5, calculated as the sales price of $35 less the exercise price of $30.
All of the following are examples of a defined contribution plan EXCEPT: Question 18 options: A) Stock Bonus Plans B) ESOPs C) Thrift Plans D) Cash Balance Pension Plans
D) Cash Balance Pension Plans Answer choices (A), (B), (C) are correct. Cash Balance Plans are NOT a type of defined contribution plans and are instead classified as a type of defined benefit plan.
In 2021, Chip, an accomplished professional race car driver, is to receive a signing bonus for agreeing to drive for Hot-Lap International, a racing team. Hot-Lap agrees to establish a NQDC agreement with Chip to defer the bonus beyond Chip's peak income producing years. Hot-Lap transfers the bonuses to an escrow agent, subject to the risk of forfeiture to team creditors in bankruptcy, who invests the funds in securities acting as a hedge against inflation. The bonus is deferred until 2023 and is then paid to Chip in years 2024-2030. When is the income deductible by the employer and includible by Chip? Option - Employer Deduction - Employee Inclusion A. 2021 2021 B. 2022 2022 C. 2023-2030 2023 D. 2024-2030 2024-2030
D) D The income is only deductible when it is includible by Chip in his income from 2024- 2030.
Robbie is the owner of SS Automotive and he would like to establish a qualified plan. Robbie would like to maximize contributions as he feels behind in his retirement savings and he loves the idea of tax deductions. He also wants to benefit the employees who have been with him over the years and he wants a long vesting period so they have an incentive to stay. Robbie is 58 and earns $700,000 per year. His employees are aged 25, 29, and 32, and they each earn $25,000 per year. Which of the following qualified pension plans would you recommend that Robbie establish? Question 53 options: A) Target-benefit pension plan B) Cash-balance pension plan C) Money-purchase pension plan D) Defined-benefit pension plan
D) Defined-benefit pension plan The target-benefit plan and the money-purchase pension plan will not reward past service which is one of Robbie's goals. Both plans are defined contribution plans and are limited by 415(c) at 25% covered compensation. Defined benefit plans do not operate under that restriction. Lastly, the Cash Balance Pension Plan will not satisfy Robbie's goals as they require 100% vesting in 3 years.
Which of the following situations would create an inclusion in an employee's gross income? Question 86 options: A) Kay is the director and manager of Holiday Hotel. As a condition of her employment, Kay is required to live at the hotel. The value of this is $1,000 per month. B) Nate works at a fitness club. The club posts favorable comments about him on social media, and this increases Nate's private massage business by at least $500 in revenue each year. C) Brian is an airline pilot with We Don't Crash Airlines, Inc. and is allowed to fly as a passenger for free on the airline whenever an open seat is available. D) Eric, who is an active duty Marine, moved from Houston to New Orleans as was required by the Marines. His expenses for the move included $400 of truck rental costs, $100 of lodging and $200 of pre-move house hunting expenses. Eric's employer reimbursed him $600.
D) Eric, who is an active duty Marine, moved from Houston to New Orleans as was required by the Marines. His expenses for the move included $400 of truck rental costs, $100 of lodging and $200 of pre-move house hunting expenses. Eric's employer reimbursed him $600. Eric must include $100 of the reimbursement in his gross income as pre-move house hunting expenses are not qualified moving expenses. To the extent that an individual is reimbursed for non-qualifying expenditures, the individual must include the reimbursement in his taxable income. In this case, the lodging of $100 and the truck rental of $400 are qualified expenses, so any reimbursement in excess of $500 ($400 + $100) is included in Eric's gross income. Since Eric's employer reimbursed him $600, $100 is taxable income to him. In option (A), Kay is required to live at her employer's hotel as a condition of employment, thus the provision of the housing is not taxable. Option (B) describes s a benefit that is only theoretical in value. Option (C) describes a "no additional cost benefit" that is not taxable to Brian. After 2017, moving expenses are not deductible except for active duty members of the Armed Forces.
Which of the following qualified plan distributions are subject to a 10% early withdrawal penalty? I. Carolyn, aged 56, is currently employed by UBEIT Corporation and takes a $125,000 distribution from the UBEIT 401(k) plan.II. Brad, aged 60, takes a $1,000,000 distribution from his employer's profit-sharing plan. Ten days after receiving the $800,000 check (reduced for 20% withholding), Brad deposited the $800,000 into a new IRA account.III. Tara, aged 22, withdraws $2,000 of her contributions from her 401(k). Question 46 options: A) I only B) III only C) II and III D) I and III
D) I and III Statement I is subjected to the 10% early withdrawal penalty, because Carolyn has not separated from service. Statement II will not be subjected to the 10% early withdrawal penalty because Brad is older than 59-1/2 years old. Statement III is subjected to the 10% penalty, because Tara does not qualify for any of the exclusions from the 10% penalty.
Which of the following objectives are considered by employers when determining which retirement plan to provide employees of their company or institution? I. Benefits to the employer II. Benefits to all employees III. Benefits to select employees IV. Tax benefits to the employer and employees Question 27 options: A) I, III, IV B) I, II, III C) II, III, IV D) I, II, III, IV
D) I, II, III, IV All four of these issues can be plan objectives considered by employers in selecting a retirement plan for their business or institution.
Organic, Inc. sponsors a qualified plan that requires employees to complete one year of service and to be 21 years old before entering the plan. The plan also excludes all commissioned sales people and all other allowable exclusions permitted under the code. Which of the following employees could be excluded? I. Sarah, age 32, who has been a secretary for the company for 11 monthsII. Andy, age 20, who works in accounting and has been with the company for 23 monthsIII. Erin, a commissioned sales clerk, who works in the Atlanta office. Erin is 25 years old and has been with the company for 4 years.IV. George, age 29, who works in the factory. George has been with the company for 9 years and is covered under a collective bargaining agreement. Question 14 options: A) I only B) I and III C) I, II, and IV D) I, II, III, and IV
D) I, II, III, and IV Each of these employees can be excluded from the plan. Sarah does not meet the service requirement. Andy does not meet the age requirement. Erin can be excluded because she is a commissioned salesperson. George is excluded because he is covered under a collective bargaining agreement.
Professor Ramiro contributes $85,000 to one of his university "retirement type" plans. He is not sure of the differences between the plans. What type of plan did he contribute the funds to? A) Tax sheltered annuity. B) Public 457 plan. C) Private 457 plan. D) Ineligible 457 plan.
D) Ineligible 457 plan. The only plan that Professor Ramiro could contribute that amount of money is a 457(f) plan, which is an ineligible 457 plan.
Child Assist, Inc. has a written adoption assistance program that pays adoption expenses including attorney and other fees, as well as other normal expenses. Child Assist paid adoption expenses for the following employees during 2023. Employee and Age - Employee AGI - Adoptee and Age -Amount Paid -Health of Adoptee Joe (34) $150,000 Cindy Lou (18) $15,000 Excellent James (34) $290,000 Randi (6) $15,000 Excellent Donna (34) $80,000 Brooke (3) $15,000 Excellent Connie (32) $100,000 Silky (5) $15,000 Excellent Who must include the total amount of the adoption assistance paid as a result of participating in the employer's adoption assistance program? A) No one, as $15,950 is the maximum excludable limit for 2023. B) Joe C) James D) Joe and James
D) Joe and James Joe has adopted an 18-year-old whose health is excellent, so he must include the amount paid as income. An "eligible child" must be under the age of 18 or be physically or mentally incapable of caring for himself. James is over the top-end of the income phaseout ($279,230) and must include the $15,000 in income.
All of the following statements concerning the income limits imposed for determining whether one is eligible to receive a deduction for contributions made to a traditional IRA are correct EXCEPT: A) The income limits only apply to taxpayers who are covered by a qualified retirement plan. B) The income limits are higher for Roth IRAs than for traditional IRAs. C) Taxpayers whose income is over the limit may still make non-deductible contributions to traditional IRAs. D) Marital filing status does not affect IRA income limits.
D) Marital filing status does not affect IRA income limits. The IRA income limits for married-filing-jointly taxpayers are about twice as high as the limits for single filers. All of the other statements are correct.
Lynn, aged 42, has an IRA with an account balance of $200,000 and adjusted gross income (AGI) will be $80,000 this year. She has recently been declared disabled. According to this information, which of the following statements is (are) true? I. She can immediately borrow up to $50,000 from her IRA account and repay the loan within 5 years. II. If she takes a $50,000 distribution from her IRA, only a portion would be subject to the 10% early withdrawal penalty. Question 33 options: A) I only B) II only C) Both I and II D) Neither I nor II
D) Neither I nor II Statement I is incorrect because loans are not permitted from IRAs. Statement II is incorrect because her declared disability permits her to avoid the penalty on the entire distribution.
Which of the following statements is (are) correct regarding assets reverting back to the sponsor or a qualified plan? I. Under a merger, assets from a qualified plan can revert back to the plan sponsor without regard to the relationship between the value of the plan assets compared to the value of the obligations under the plan.II. Any reversion of plan assets will only be subject to a 20-percent penalty. Question 58 options: A) I only B) II only C) Both I and II D) Neither I nor II
D) Neither I nor II Statement I is incorrect because the assets must exceed the plan liabilities. Statement II is incorrect because the penalty may be 20 percent or 50 percent.
One of the disadvantages of an ESOP is that the stock is an undiversified investment portfolio. Which of the following is correct? I. An employee, aged 55 or older, who has completed 5 years of participation in an ESOP may require that up to 25 percent of the account balance be diversified.II. An employee who receives corporate stock from an ESOP in a series of periodic distributions over 3 years may enjoy net unrealized appreciation treatment at the time of each distribution. Question 37 options: A) I only B) II only C) Both I and II D) Neither I nor II
D) Neither I nor II The first statement is incorrect because the time in the plan required to request diversification is 10 years not 5. The second statement is incorrect because net unrealized appreciation treatment is only available in the case when all the account assets are distributed within one tax year.
The High-End Preparatory Boarding School has a residence on its property for the principal. Jacques, who is the principal, and his family live in the residence, which is provided as part of his job. The school's board of directors consider it important that the principal be close to the school in case any issues arise. Jacques also eats at the school cafeteria, typically two meals each school day. Which of the two benefits must be included in Jacques' gross income for tax purposes? I. Lodging II. Meals A) I only B) II only C) Both I and II D) Neither I nor II
D) Neither I nor II The meals and lodging are provided for the convenience of the employer. After 2017, the TCJA 2017 subjects the meals for the convenience of the employer to the 50-percent deduction limit.
Which of the following statements regarding an age-based profit-sharing plan is correct? Question 31 options: A) An age-based profit-sharing plan only provides a benefit to those plan participants whose age is within 10 years of the age of the owner of the plan sponsor. B) An age-based profit sharing plan provides a greater benefit to those plan participants who are over fifty years old and whose earnings exceed the Social Security wage base. C) The majority of the profit-sharing allocation in an age-based profit-sharing plan is usually received by younger plan participants. D) Older plan participants receive greater benefits from an age-based profit-sharing plan.
D) Older plan participants receive greater benefits from an age-based profit-sharing plan. An age-based profit-sharing plan provides a greater benefit to older plan participants because the allocation of the plan contribution is based upon the age of the participants. Options (A) and (B) are false statements without any merit. Option (C) is incorrect because OLDER plan participants in an age-based profit-sharing plan usually receive the majority of the profit-sharing allocation.
Acme Corporation has created a deferred compensation plan (Plan) for 50 key executives (participants), all of whom are highly compensated employees. Acme contributes each year on behalf of each participant to a trust, T. The trust is not and never has been a qualified trust under §401(a) and is not exempt from taxation under §501(a). T's assets are not subject to the claims of Acme's creditors. Separate accounts that reflect the participant's share of the net trust assets and income are maintained for each participant. A participant's entire interest in T becomes vested upon completion of three years of service with Acme beginning on the date the individual first becomes a participant in the plan. Participants or their beneficiaries are entitled to receive their vested interest in the net assets of T, net of applicable withholding and other taxes, on death, disability, or termination of employment. In addition, T
D) Secular trust A rabbi trust would be subject to claims of creditors in a bankruptcy. It also does not distribute money for taxes.
Permitted qualified distribution options from qualified plans include all of the following EXCEPT: Question 22 options: A) Annuities (Single, Joint Life, Joint and Survivor, etc.) B) Lump Sum C) In-service Withdrawals D) Tax-Free Non-Repaid Loans
D) Tax-Free Non-Repaid Loans Non-repaid loans are a taxable distribution from qualified plans. (A), (B) and (C) are all permitted distributions from qualified plans.
All of the following are problems associated with Monte Carlo Analysis EXCEPT: Question 8 options: A) Means and standard deviations for stock returns vary. B) It assumes normal distribution and serial independence for investment returns. C) Many Monte Carlo Analysis calculators ignore income tax consequences. D) The analysis only provides the most likely outcome.
D) The analysis only provides the most likely outcome. While a Monte Carlo Analysis has the problems listed in (A) through (C), the very advantage of the analysis is that is provides the most likely outcome and, typically, other possible outcomes.
Wallace and Associates is considering implementing a buy/sell agreement in which each partner purchases a life insurance policy on each of the other partners. Which one of the following statements is correct, given this information? Question 99 options: A) The partners are entering into an entity redemption agreement. B) Upon the death of an owner, the life insurance proceeds will be used to buy out the decedent's share of the partnership. Those life insurance proceeds are taxable as ordinary income. C) The amount of insurance per policy will equal the value of the partnership. D) The partners are entering into a cross-purchase agreement.
D) The partners are entering into a cross-purchase agreement. Statement (A) is incorrect because the entity approach occurs when the business entity purchases life insurance policies on each owner. Statement (B) is incorrect because the life insurance proceeds are tax-exempt. Statement (C) is incorrect because each policy will be for the deceased partner's share.
Which of the following statements concerning prohibited transactions is true? Question 59 options: A) The plan is free to lend its assets to company owners. B) The CPA auditor of the plan is never a disqualified person. C) The tax on prohibited transactions is 25% of the amount involved. D) The plan's investment advisor is a disqualified person.
D) The plan's investment advisor is a disqualified person. Any person providing services to a qualified plan is a disqualified person. Statement (A) is incorrect because a plan is prohibited from lending to a disqualified person. Statement (B) is incorrect because a plan service provider is always disqualified. Statement (C) is incorrect because the excise tax on prohibited transactions is 15% in the year of the transaction, plus an additional 100% if the transaction is not corrected within the taxable period.
To qualify for non-recognition of gain treatment, all of the following requirements apply EXCEPT: Question 39 options: A) The ESOP must own at least 30% of the corporation's stock immediately after the sale. B) The corporation that establishes the ESOP must have no class of stock outstanding that is tradeable on an established securities market. C) The seller and 25% shareholders in the corporation are precluded from receiving allocations of stock acquired by the ESOP through the rollover. D) The stock sold to the ESOP must be common or convertible preferred stock and must have been owned by the seller for no more than three years prior to the sale.
D) The stock sold to the ESOP must be common or convertible preferred stock and must have been owned by the seller for no more than three years prior to the sale. The stock sold must have been owned by the seller for at least 3 years. All other statements are correct.
All of the following are characteristics of de minimis fringe benefits EXCEPT: Question 44 options: A) They can be discriminatory. B) They are infrequently offered. C) Accounting for them is deemed to be impractical. D) There are no exceptions for cash as a de minimis benefit.
D) There are no exceptions for cash as a de minimis benefit. Answer choices (A), (B), and (C) are all characteristics of de minimis fringe benefits. Answer choice (D) is incorrect as a statement in that normally cash benefits would be includable in compensation except where the cash is for reasonable, occasional meals or transportation fare.
Alfred has worked for CJD, a large manufacturer, for the last 20 years and is a participant in CJD's defined-benefit plan. Alfred is concerned about the company's financial difficulties and is worried that management at CJD might modify his future benefits and cause his expected benefits at retirement to be reduced. Which of the following laws is designed to prevent that from occurring? A) Coverage B) Non-discrimination C) Anti-cutback D) There are no laws that prevent an employer from modifying future benefits, even benefits in a defined-benefit plan.
D) There are no laws that prevent an employer from modifying future benefits, even benefits in a defined-benefit plan. Future benefits can be modified by employers. Coverage is designed to ensure that a minimum number of non-highly compensated employees are benefiting under the plan. Non-discrimination rules do not prevent loss of benefits. Anti-cutback is a rule prohibiting a plan sponsor from amending a plan such that the accrued benefit of an employee is decreased or reduced by such an amendment or change.
Which of the following are characteristics of all profit-sharing plans? Question 14 options: A) They require annual plan contributions. B) Compared to other qualified plans, they provide employee participants the greatest certainty when funding their own retirement. C) The amount that an employer does or doesn't contribute to a profit-sharing plan is based solely on the plan document filed with the IRS. D) They can allow for distribution of stock or cash, depending on the complexion and requirements of the plan.
D) They can allow for distribution of stock or cash, depending on the complexion and requirements of the plan. Statement (A) is incorrect because neither employees nor employers are required to make annual contributions. Statement (B) is incorrect because defined benefit plans provide far more certainty in regards to retirement funding. (C) is incorrect in that the amount an employer does or doesn't contribute to a profit-sharing plan is a business decision and not based on the plan document. The formula delineating how much will be pares out to the participant's account, will be included in the plan documents.
Bob, who is 58 years old, makes his first contribution to a Roth IRA in December of Year 1. What is the first year in which he could receive a qualified distribution from a Roth IRA? A) Year 2 B) Year 3 C) Year 5 D) Year 6
D) Year 6 A qualified distribution can only occur after a 5-year period and is made on or after the date on which the owner attains age 59½, made to a beneficiary or the estate of the owner on or after the date of the owner's death, attributable to the owner's being disabled, or made for a first-time home purchase. The 5-year period begins at the beginning of the taxable year of the initial contribution to a Roth IRA. The 5-year period ends on the last day of the individual's fifth consecutive taxable year beginning with the taxable year described in the preceding sentence. Therefore the first year in which a qualified distribution could occur is Year 6.
All of the following are common defined-benefit plan funding formulas EXCEPT Question 20 options: A) the flat amount formula. B) the flat percentage formula. C) the unit credit formula. D) the excludable amount formula.
D) the excludable amount formula. Answers (A), (B), and (C) are common defined-benefit-plan funding formulas. There is no such thing as an excludable amount formula.
When considering benefit payouts to participants in a failed plan administered by the Pension Benefit Guaranty Corporation (PBGC), which of the following would be taken into consideration? Question 51 options: A) whether the participant was a key employee. B) the participant's health status. C) the participant's ownership status. D) the participant's age.
D) the participant's age The payout type, participant's age, and degree to which the terminated plan was underfunded all factor into the benefit calculation.
Which of the following people can make a deductible contribution to a traditional IRA for 2023? Person Adjusted Gross Income (AGI) Covered by Qualified Plan Marital Status Devon $100,000 Yes Married Gretchen $50,000 Yes Single Destiny $280,000 No Married Aria $179,000 Yes Single
Devon, Gretchen and Destiny All but Aria may deduct a contribution made to a traditional IRA. Devon and Gretchen are below the phase-out range. Destiny is not covered by a qualified plan, so there is no income limit. Aria is single and covered by a plan, and her AGI is above the top end of the phaseout for singles ($73,000-$83,000 for 2023).
Which of the following benefits provided by an employer to its employees is currently taxable to the employee? Employees of the KM Department Store are allowed a 15 percent discount on store merchandise. KM's normal gross profit percentage is 20 percent. On a space-available basis, undergraduate tuition is waived by Secluded University for the dependent children of employees (value of $15,000 per semester). Apollo Airlines allows its employees to fly free when there are open seats available on a flight (average value of $200). Employees of Indigo Associates are allowed incidental personal use of a company car.
Employees of Indigo Associates are allowed incidental personal use of a company car. Personal use of a company car is a taxable fringe benefit. All of the other fringe benefits listed may be excluded from the employee's gross income.
One of the purposes of a retirement plan is to accumulate retirement assets. Increases in retirement assets are a result of contributions by employers and employees as well as investment earnings on the retirement assets. Which of the following is correct regarding the investment management of retirementassets? A) The primary reason that employees are given the right to self-direct plan assets is to empower them. B) Employers can commingle assets from employees' retirement accounts and have the assets manged by a third-party asset manager or manage the assets in-house. C) Employers eliminate fiduciary responsibility by allowing employees to self-direct their retirement accounts. D) The employer cannot manage assets in a 401(k) plan.
Employers can commingle assets from employees' retirement accounts and have the assets manged by a third-party asset manager or manage the assets in-house. Option A is false because the right given to employees to self-direct plan assets is to reduce the employer's liability. Option C is false because it does not fully eliminate liability or responsibility. Employers must still select purdent investment options for the plan. Option D is false as the employer can manage the assets in a 401(k) plan.
Which of the following statements regarding determination letters for qualified plans is true? A) Employers must request a determination letter from the IRS for all qualified plans. B) Employers must request a determination letter from the DOL for all qualified plans that are materially modified or materially amended. C) Employers who receive a favorable determination letter are protected from the IRS disqualifying their qualified plan. D) Employers must follow each and every aspect of their qualified plan document.
Employers must follow each and every aspect of their qualified plan document. Determination letters are issued by the IRS at the request of the plan sponsor. The plan sponsor is not required to request a determination letter. Even if the determination letter is requested and approved, the IRS may still disqualify the plan. Employers must follow their plan.
Randolph is retiring early from his position as CEO of BIG Co. BIG is providing him with a lucrative severance package for early retirement. Which of the following is the name of this type of benefit? A) Golden Handcuff. B) Golden Parachute. C) Golden Retriever. D) Golden Handshake.
Golden Handshake. Handcuffs keep the executive with the company. Parachute is from termination in corporate ownership change. Golden Retriever is a dog.
Which of the following statements concerning health savings accounts (HSAs) is correct? HSA distributions used to pay for nonmedical expenses are subject to a 10 percent penalty. HSA distributions not used for medical expenses are subject to federal ordinary income taxes. There is no limit to the amount an individual may contribute to an HSA each year. A married couple may contribute to up to one medical flexible spending account (FSA) and up to one HSA in any given year.
HSA distributions not used for medical expenses are subject to federal ordinary income taxes. HSA distributions not used for medical expenses are subject to federal ordinary income taxes and a 20 percent penalty (not, as the first option states, a 10 percent penalty). The third option is incorrect because HSA contributions are subject to an annual inflation-adjusted maximum. The fourth option is incorrect because if one spouse is covered by an FSA, it is treated as both spouses being covered. If eligible, a married couple must select between an FSA and an HSA unless the FSA is limited purpose (covers dental and vision only).
Haley, age 25, and her spouse, who is active duty military, recently moved to Fayetteville, NC due to a military permanent change of station (PCS). Haley is excited to begin her new job working for a small employer who offers a defined contribution profit sharing plan. The plan follows the standard eligibility rules with a three-year cliff vesting schedule, and has elected to follow the new SECURE 2,0 Act rules for accelerated eligibility and vesting for military spouses. Haley's salary will be $130,000. Which of the following Question 1 options: A) Haley will be eligible to participate after one year of service and will be fully vested after three years. B) Haley will be eligible to participate after two years of service and will be fully vested immediately. C) Haley will be eligible to participate two months after her date of hire and will be immediately vested. D) Haley will be eligible to participate two months
Haley will be eligible to participate two months after her date of hire and will be immediately vested. Under the SECURE 2.0 Act, small employers offering a defined contribution plan receive a tax credit to help offset the additional cost of offering accelerated eligibility and vesting to military spouses. To qualify, the military spouse must be eligible to participate within two months after their date of hire, and must be immediately vested in all employer contributions to the plan.
Which of the following plans generally favor(s) older participants? I. Defined-benefit pension plans II. Cash-balance pension plans III. Target-balance pension plans IV. Money-purchase pension plans I and II I and III II and III IV only
I and III Cash-balance and money-purchase pension plans generally favor younger participants, while defined-benefit and target-benefit pension plans favor older participants with less time to accumulate and require higher funding levels.
The early distribution penalty of 10 percent does NOT apply to qualified plan distributions that are: I. made after attaining age 55 and then separating from service. II. made for the purpose of paying qualified higher education costs. III. paid to a designated beneficiary after the death of the account owner who had not begun receiving minimum distributions. I only I and III II and III I, II, and III
I and III Statement II is an exception for distributions from IRAs, not qualified plans. Statements I and III are exceptions to the 10% penalty for qualified plan distributions.
Which of the following is (are) an element(s) of an effective waiver for a pre-retirement survivor annuity? I. Both spouses must sign the waiver. II. The waiver must be notarized or signed by a plan official. III. The waiver must indicate that the person(s) waiving understand the consequences of the waiver. I only I and III II and III I, II, and III
II and III Only the nonparticipant spouse must sign the waiver.
A business valued at $3,000,000 has three partners. Each of the three partners buys a $500,000 life insurance policy for purposes of a buy-sell agreement on each of the other partners. Which of the following statements is (are) true? I. This is an example of an entity-purchase plan. II. This is an example of a cross-purchase plan. III. The buy-sell agreement is underfunded. I only II only I and III II and III
II only This is a cross-purchase life insurance plan. Each person has a one-third interest. Therefore when the first partner dies, the other two partners will each need to pay $500,000 for a total of $1,000,000 (⅓ of $3,000,000). Thus the buy-sell agreement is not underfunded.
Which of the following statements concerning money-purchase pension plans is (are) correct? I. Businesses with cash-flow problems are especially good candidates for a money-purchase pension plan. II. Money-purchase pension plans have the advantages of predictable cost for the employer and understandability for the employees. I only II only Both I and II Neither I nor II
II only With a money-purchase pension plan, the employer (the plan sponsor) does not bear any investment risk; therefore, the employer's costs will be predictable. I is incorrect because the employer is required to fund a money-purchase pension plan annually, which requires a steady cash flow.
George, age 40, is covered by a $100,000 face value term life insurance policy as part of profit-sharing plan at his place of employment. How much taxable income will be attributed to George this year?
IRS Table 2001 shows $1.10 per $1,000 of life insurance protection. Taxable income is $1.10 x ($100,000 / $1,000) = $110.
Roberta works for Microsoft, a publicly traded company that sponsors a stock bonus plan. Which of the following statements is not correct regarding the plan? A) If Roberta has less than three years of service, she is permitted to diversify one-half of company match contributions that consist of Microsoft stock. B) Roberta is permitted to vote the shares of Microsoft within her account. C) Upon termination, Roberta must be given the right to receive Microsoft stock held in the plan as part of a distribution. D) If the distribution of Microsoft stock is made to Roberta in installments over a two-year period, then the fair market value of all employer securities distributed in the installment distribution will be taxable as ordinary income.
If Roberta has less than three years of service, she is permitted to diversify one-half of company match contributions that consist of Microsoft stock. After three years of service, employer contributions of company stock can be fully diversified. Before three years, employees do not have the right to diversify company stock in a stock bonus plan of a publicly traded company. The other statements are correct.
Sarah works for K Corp. On January 3, 2022 she was granted ten restricted shares of K Corp's stock, which was valued at $10 per share. The restrictions are to come off in 3 years (2025). On January 14, 2022 Sarah filed a Section 83(b) notice with the IRS. Which of the following statements is correct? If K Corp goes bankrupt in 2023 and is liquidated, Sarah cannot deduct the loss she incurred. If Sarah takes no action with her restricted stock after filing the Section 83(b) election and retains the stock, she will not incur any ordinary income in 2025 when the stock vests. Sarah would pay ordinary income tax on the gain in her stock when she sells her stock in 2024. Because this is not a nonqualified stock option (NQSO), Section 83(b) is not available.
If Sarah takes no action with her restricted stock after filing the Section 83(b) election and retains the stock, she will not incur any ordinary income in 2025 when the stock vests. By making a Section 83(b) election, she agrees to have the stock taxed currently (2022) at its current value ($10 per share) even though the stock is restricted. When it vests, there's no immediate tax effect, and when she sells her stock, it will be subject to capital gains over her $10-per-share basis.
If Sergei selected the standard allocation method and the plan contributed 10 percent per individual, the plan would contribute $50,000 to Sergei's account. If Sergei selected the permitted disparity method as a profit-sharing formula, the total contribution the company makes for Sergei would be increased. Considering the needs and wants of Sergei and Kahlil, an age-based profit-sharing plan would be the best plan for both. A new comparability plan would be the least expensive and simplest way to meet both Sergei and Kahlil's retirement needs.
If Sergei selected the permitted disparity method as a profit-sharing formula, the total contribution the company makes for Sergei would be increased. By using permitted disparity, or Social Security integration, and assuming that the integration level is set at the taxable wage base (TWB), Sergei can effectively increase contributions for himself as Kahlil and the rest of the employees do not make enough to exceed the integration level (TWB). (A) is false because the contribution to Sergei's account using a standard allocation of 10 percent would not exceed the 415(c) compensation limit for that year. Sergei earns well in excess of that. (C) is false because an age-based profit sharing is not necessarily in Kahlil's best interest. The facts state that the 15 other employees range from age 25 to age 35, with some of these employees being older than Kahlil. In this instance, some employees might be allocated a greater share of the contribution than Kahlil. (D) is false because a new comparability plan would generally be more expensive to design and administer than other plans. One final note, for plan design and administration considerations, it must be remembered that Khalil will be deemed to own all the shares that Sergei owns due to family attribution.
Which of the following statements concerning the typical Voluntary Employees' Beneficiary Association (VEBA) is correct? Membership is mandatory for all employers that provide employee benefits. Employers may discriminate in favor of highly compensated employees. Certain benefits, such as vacations and job training benefits, are not covered by a VEBA. If an employer participates in a VEBA, there is still a chance for an employee's benefit to be underfunded.
If an employer participates in a VEBA, there is still a chance for an employee's benefit to be underfunded. While a VEBA reduces the likelihood that an employee benefit is underfunded, this risk is not fully eliminated. A VEBA relies on actuarial data to anticipate future costs, but such predictions are not perfectly accurate. The first option is incorrect because membership to a VEBA is voluntary. The second option is incorrect because employers may not discriminate in favor of highly compensated employees. The third option is incorrect because, while certain benefits are not covered by a VEBA, both vacations and job training benefits are covered.
Jack and Jill own a successful engineering company that sponsors a 401(k) plan that requires standard eligibility. Sam, Tom, and Pat are the only other employees. These employees are between the ages of 25 and 29 and have been with the company for a couple of years. Jack and Jill each have salaries of $200,000, while their employees have salaries ranging between $28,000 and $30,000. Jack and Jill both defer $10,000 each. Sam, who is Jack and Jill's son, earns $30,000 and defers $6,000 into the 401(k) plan. Tom, who makes $28,000, defers $2,800, while Pat does not defer anything into the 401(k) plan. Which of the following statements is correct? A) The ADP for the non-highly compensated employees (NHCEs) is 6.67 percent. B) Jack and Jill are the only highly compensated employees (HCEs). C) If the plan failed the ADP test, the issue could be solved by providing a qualified matching contribution (QMC) to all five emplo
If the company hired a new employee, it would not increase the amount that Jack and Jill can defer during the first year of the employee's employment. Sam is highly compensated through family attribution. The ADP of the NHCEs is 5 percent, not 6.67 percent. Statement (C) is not correct as a QMC would only go to NHCEs.
All of the following statements concerning IRAs are correct, EXCEPT: Contributions can be made to the plan after a participant has attained age 72. In most cases, an individual can contribute both the maximum contribution to a Roth IRA and the maximum contribution to a traditional IRA for a given tax year. Whether deductible or nondeductible contributions were made, Traditional IRAs are still subject to RMDs. Contributions to a Roth IRA are never deductible.
In most cases, an individual can contribute both the maximum contribution to a Roth IRA and the maximum contribution to a traditional IRA for a given tax year. If the maximum is contributed to a Roth IRA, no contribution can be made to a traditional IRA maintained on behalf of the same taxpayer.
All of the following statements are correct regarding 403(b) plans, EXCEPT: 403(b) plans are eligible for rollover treatment to IRAs, qualified plans, and other 403(b) plans. Investments in stocks, bonds, and money markets are available. Assets in a 403(b) plan are generally 100 percent vested. Typically, 403(b) plans have a lower threshold of entry than do 401(k) plans.
Investments in stocks, bonds, and money markets are available. The second option is incorrect because investments are limited to insurance annuities and mutual funds. All of the other statements are correct.
Johnny is 35 years old and inherits an IRA from his mother, who dies prematurely at age 60 in May 2023. Which of the following statements is correct regarding his options for the inherited IRA? A) Johnny does not have to take distributions until he turns 75 years old. B) Johnny can rollover the IRA into his own IRA. C) Johnny must take out the entire account value within 10 years. D) Johnny can take distributions over his single life expectancy.
Johnny must take out the entire account value within 10 years. Option A is not correct. Option B is available to surviving spouses, but not a child. Option D is no longer correct since his mom died after 2019. Under the SECURE Act, for deaths after 2019, ineligible beneficiaries must take full distribution within 10 years of death. Only eligible beneficiaries (surviving spouse, minor child of the decedent, disabled beneficiaries, and beneficiaries no more than 10 years younger than the decedent) are permitted to take distributions over their life expectancy.
Kelli works for J Corp. In January 2022, she was granted ten restricted share units at J Corp's current value of $10 per share. The shares will not vest until 3 years later, that is, in January 2025. In December 2025, Kelli sells all her J Corp shares for $30 per share. Which of the following statements about the tax effects of this transaction is correct? Kelli will include the fair market value (FMV) of the shares in her gross income in 2025. Kelli will include the fair market value (FMV) of the shares in her gross income in 2022. Kelli's income tax basis in the stock will be $20 per share upon vesting in 2025. Kelli's income tax basis in the stock will be $10 per share at the time of grant in 2022.
Kelli will include the fair market value (FMV) of the shares in her gross income in 2025. When the stock's substantial risk of forfeiture is removed (that is, when Kelli becomes vested), the shares are taxed to Kelli as ordinary income. That will also serve as her tax basis in the stock.
Laurent owns Marketing Solutions, Inc., (MSI) and sells 100 percent of the company stock on July 1 of the current year to an employee stock ownership plan (ESOP) for $3,000,000. Laurent had an adjusted basis in the MSI stock of $450,000. If Laurent reinvests in qualified replacement securities before the end of the current year, which of the following statements is true? Laurent will not recognize long-term capital gain or ordinary income in the current year. Laurent must recognize $2,550,000 of long-term capital gain in the current year. Laurent must recognize $450,000 of ordinary income in the current year. If Laurent dies before selling the qualified replacement securities, his heirs will have an adjusted taxable basis in the qualified replacement securities of $450,000, Laurent's carryover adjusted basis.
Laurent will not recognize long-term capital gain or ordinary income in the current year. A major advantage for an ESOP is the ability of the owner to diversify his interest in a closely held corporation. In this case, if Laurent reinvests in qualified replacement securities within 12 months of the sale to the ESOP, he will not recognize capital gain or ordinary income on the sale to the ESOP. If Laurent dies, the heirs will receive the securities with an adjusted taxable basis equal to the fair market value (FMV) at Laurent's date of death or the alternate valuation date.
Shireen is a fulltime employee of Exgone Inc. She is offered participation in a Sec. 423 employee stock purchase plan (ESPP). All of the following are reasons she should consider participating in the ESPP EXCEPT She will not have to recognize income tax on the value of the stock versus the purchase price at the time she acquires the stock The effective discount can be higher than 15% off of the stock's price. Like nonqualified stock options, Shireen will be able to defer tax until she exercises the option, and at that point pay ordinary income tax Shireen can purchase the stock through the ESPP less expensively than if she purchased the stock on the open market.
Like nonqualified stock options, Shireen will be able to defer tax until she exercises the option, and at that point pay ordinary income tax All the other statements are true. With (B), even though the discount is limited to 15%, the effective discount can be much higher. (C ) is incorrect both because an ESPP is not a stock option, but rather a stock purchase, and also because she will not pay ordinary income tax on all of the gain.
The early distribution penalty of 10 percent applies to IRA distributions that are Made after the plan participant attains age 55 and is separated from service. Made for the purpose of paying qualified higher education costs. Paid to a designated beneficiary after the death of the account owner, who had not begun receiving minimum distributions. Made after the onset of a disability.
Made after the plan participant attains age 55 and is separated from service. There is a 10 percent penalty for IRA distributions made before 59½ years of age. The first option would be excepted from that penalty if it were made from a qualified plan, but it is not an exception for distributions from an IRA. The other answers are exceptions to the 10 percent penalty for IRAs.
Which of the following is a characteristic of pension plans that does not apply to profit-sharing plans? Question 2 options: A) Separate accounts B) In-service withdrawals for select employees, plan permitting C) Limited investment in life insurance D) Mandatory funding
Mandatory funding A pension plan requires mandatory funding. In-service withdrawals may be offered in a profit-sharing plan. A pension plan may allow in-service withdrawals for employees over the age of 59 1/2. Both pension and profit-sharing plans have restrictions on the amount of life insurance that can be purchased within the plan.
All of the following statements are correct EXCEPT: Defined benefit and cash balance pension plans require the use of an actuary to estimate plan funding requirements. Target benefit pension plans require the use of an actuary at the inception of the plan only. Money purchase pension plans require the use of an actuary at the inception of the plan only. Money purchase pension plans do not require the use of an actuary at all.
Money purchase pension plans require the use of an actuary at the inception of the plan only. Money purchase pension plans do not require the use of an actuary given that they fall under the category of being a defined contribution plan.
The actual deferral percentage (ADP) test is one of the tests that some qualified plans must comply with. Which of the following plans must generally comply with the ADP test? Profit-sharing plans. Safe harbor 401(k) plans. A) 1 only. B) 2 only. C) Both 1 and 2. D) Neither 1 nor 2.
Neither 1 nor 2. The ADP test generally only applies to non-safe harbor 401(k) plans. It does not apply to profit-sharing plans without a CODA.
Which of the following is correct with regard to adoption assistance programs? I. The amount paid is excluded from the employee's income regardless of the employee's income level. II. The unlimited amount paid is excluded from the employee's income with a phaseout of adjusted gross income (AGI) at certain levels. I only II only Both I and II Neither I nor II
Neither I nor II Statement I is false because there is a limit on the expense paid and an AGI phaseout of $239,230 through $279,230 for 2023. Statement II is false because there is a limit on the expense for adoption assistance programs.
Generally, distributions from a retirement plan are subject to income tax as ordinary income. Which of the following tax treatments is not an exception to ordinary income on a distribution from a qualified plan? A) 401(k) loan. B) Non-recognition of gain treatment. C) IRA rollovers. D) Net unrealized appreciation.
Non-recognition of gain treatment. Non-recognition of gain treatment is the deferral of capital gain (not ordinary income) that has nothing to do with a distribution from a qualified plan.
Which of the following statements about stock bonus plans is false? A) A CODA can be attached to a stock bonus plan. B) Participants of a stock bonus plan must have the right to demand employer securities on plan distributions. C) Stock bonus plans have a deductible contribution limit of 25 percent of compensation. D) None of the above are false. All are true.
None of the above are false. All are true. Options A, B and C are all true statements.
Which of the following are correct regarding SIMPLEs? A) An employer can maintain a SEP and a SIMPLE at the same time covering the same employees. B) A SIMPLE is predominantly an employer contribution plan. C) Once deposited in a SIMPLE IRA, funds can be distributed by the participant. D) Employers must provide a matching contribution to employees covered by a SIMPLE.
Once deposited in a SIMPLE IRA, funds can be distributed by the participant. SIMPLEs cannot be maintained with any other plans. However, under the SECURE 2.0 Act, for plan years beginning after December 31, 2023, an employer sponsoring a SIMPLE IRA plan may elect at any time during the plan year to terminate the SIMPLE IRA and immediately replace it with a SIMPLE 401(k), safe harbor 401(k), QACA, or starter 401(k). A SIMPLE is primarily funded with employee deferrals, not employer contributions. Employers can provide a match or a non-elective contribution.
Which of the following statements is true regarding a deferred-compensation plan that is funded using a Rabbi trust? Participants have security against the employer's unwillingness to pay. Rabbi trusts provide the participant with security against employer bankruptcy. Rabbi trusts provide a current tax deduction for participants. Rabbi trusts provide the employer with a current tax deduction.
Participants have security against the employer's unwillingness to pay. Rabbi trusts do not provide security against employer bankruptcy or a current tax deduction for the employer, but they do provide security against an employer's unwillingness to pay and tax deferral for participants. Neither employers nor participants receive a current tax deduction. Rabbi trusts do provide tax deferral for participants.
Which of the following types of income is considered earned income for the purposes of making a Roth IRA contribution? A) Dividends B) Rental income C) Capital gains D) Partnership income from a law firm
Partnership income from a law firm Dividends and capital gains are investment income. Rental income is passive. Partnership income is considered earned income if it is from active engagement of the business.
In January, Pierce, who is 75 years old, agreed in an email with his financial advisor that he wanted to take a distribution of $50,000 from his IRA and roll it over into a new IRA. His financial advisor inadvertently moved the funds into a taxable account. This mistake was discovered by the advisor at theend of the year and corrected. As a result, the $50,000 will be treated as a taxable distribution. What should Pierce do? A) Request a waiver of the 60-day rollover requirement from the IRS or use the self-certification process. B) Pay the income tax and seek relief from the financial advisor. C) Pay the income tax. D) Pay the income tax and move his funds to a new advisor.
Request a waiver of the 60-day rollover requirement from the IRS or use the self-certification process. The IRS regularly grants waivers of the 60-day rollover period in cases of a mistake by a financial advisor. Taxpayers may also qualify to use the self-certification process (see Rev. Proc. 2016-47).
XYZ covered the following employees under its qualified plan. Rob, a 4% owner and employee with compensation of $32,000. Rochelle, Rob's cousin, a commissioned salesperson with compensation of $195,000 last year (the highest paid employee). Rochelle owns 2% of the company stock. Randi,the chief operating officer, who had compensation of $160,000 last year, but was not in the top 20% of paid employees. Assuming the company made the 20% election when determining who is highly compensated, which of the following statements is correct? A) Rob and Rochelle are both key employees. B) Rochelle is a key employee, but Rob is not. C) Randi is a key employee, but not a highly compensated employee. D) Neither Rob, Rochelle, or Randi are highly compensated employees or key employees.
Rochelle is a key employee, but Rob is not. For 2023, a key employee is an employee who at any time during the plan year or prior year met one of the following definitions: A greater than 5% owner; A greater than 1% owner with compensation > $150,000 (not indexed); or An officer with compensation in excess of $215,000. For 2023 highly compensated employees are employees that are: A more than five percent owner at any time during the plan year or preceding plan year, or An employee with compensation in excess of $135,000 for the prior plan year (2022 is the lookback year; for 2023 the threshold is $150,000), and if elected, is in the top 20% of paid employees ranked as to compensation. Rob - he is not a 5% owner. Attribution does not include cousins. His compensation is too low for him to be highly compensated. Rochelle - she is not a 5% owner. Attribution does not include cousins. She meets the definition of highly compensated because her income is over $135,000 in 2022 (the look-back year) and she is in the top 20% of paid employees. She is also a key employee as she is a greater than 1% owner and has compensation in excess of $150,000. Randi- She is an officer, but not an owner. However, she does not meet the definition of highly compensated because she is not in the top 20% of paid employees. She is not a key employee because her compensation is less than the annual limit for officers.
Fred, age 30, has a small but promising business with 5 current employees. He wishes to adopt an easily managed workplace plan that would allow himself and his employees to defer modest amounts of their salary into the plan. Fred isn't interested in maximizing his own deferrals because he is plowing most of the earnings back into the business. But he is concerned about maintaining plan choice flexibility in case his business suddenly expands in size. Which of the following plans would Fred prefer to adopt? SEP IRA Profit-sharing plan with 401(k) SIMPLE IRA Target benefit pension plan
SIMPLE IRA A SEP IRA is incorrect because SEP IRA's don't allow employee salary deferrals. A Profit-sharing plan with a 401(k) is incorrect because the additional complexity to maintain a qualified plan and higher deferral limits don't match Fred's requirements. A target benefit pension plan is incorrect because target benefit pensions don't depend on employee salary deferrals.